SlideShare a Scribd company logo
1 of 32
PATHOMORPHOLOGY GENERAL
1. Histological examination of a 40 yo. man's thymus             D. II
revealed decreased share of parenchymatous gland                 E. V
elements, increased share of adipose and loose connective
tissue, its enrichment with thymus bodies. The organ's mass      11. A 22 y.o. woman has enlarged lymph nodes.
was unchanged. What phenomenon is it?                            Histologically: a lymph node contains lymphocytes,
A. Age involution                                                histiocytes, reticular cells, small and big Hodgkin's cells,
B. Accidental involution                                         multinucleated Sternberg cells, isolated foci of caseous
C. Hypotrophy                                                    necrosis. What disease are these changes typical for?
D. Atrophy                                                       A. Lymphosarcoma
E. Dystrophy                                                     B. Lymphogranulomatosis
                                                                 C. Chronic leukosis
4. A 9 m.o. child has delayed dentition, it is also out of       D. Acute leukosis
order. Upper jaw configuration is horizontal ("high"palate);     E. Lung cancer metastasis
microscopically irregular mineralization of tooth enamel,
wrinkled enamel prisms, some of them are vacuolized.             12. A patient died under conditions of cardiovascular
Predentin zone is extended; there are solitary denticles.        insufficiency. Autopsy results: postinfarction cardiosclerosis,
What disease is it?                                              myocardium hypertrophy and dilatation of its cavities,
A. Early rickets                                                 especially of its right ventricle. Liver is enlarged, its surface
B. Hypervitaminosis D                                            is smooth, incision revealed that it was plethoric, with
C. Late rickets                                                  darkred specks against the background of brownish tissue.
D. Gout                                                          Histologically: plethora of central parts of lobules;
E. Osteomalacia                                                  peritheral parts around portal tracts contain hepatocytes in a
                                                                 state of adipose degeneration. How are these liver changes
6. A patient ill with diabetes mellitus felt acute pain in his   called?
right foot. Objectively: foot thumb is black, foot tissues are   A. Pseudonutmeg liver
edematous, there are foci of epidermis desquamation,             B. Amyloidosis
stinking discharges. What clinicopathological form of            C. C. Nutmeg liver
necrosis is it?                                                  D. Liver cirrhosis
A. Infarction                                                    E. Liver steatosis
B. Sequestrum
C. Dry gangrene                                                  13. In the surgical department of a hospital there was an
D. Bedsore                                                       outbreak of hospital infecti on that showed itself in often
E. Moist gangrene                                                postoperative wound abscesses. Bacteriological examination
                                                                 of pus revealed aurococcus. What examination shall be
8. Autopsy of a man who died from influenza revealed that        conducted to find out the source of this causative agent
his heart was slightly enlarged, pastous, myocardium was         among the department personnel?
dull and had specks. Microscopical examination of                A. Microscopical examination
myocardium revealed signs of parenchymatous adipose and          B. Phagotyping
hydropic dystrophy; stroma was edematic with poor                C. Estimation of antibiotic susceptibility
macrophagal and lymphocytic infiltration, vessels were           D. Biochemical identification
plethoric; perivascular analysis revealed petechial              E. Serological identification
hemorrhages. What type of myocarditis was developed in
this case?                                                       17. Autopsy of a man who died from chronic cardiovascular
A. Purulent                                                      collapse revealed "tiger heart". Sidewards of endocardium a
B. Granulomatous                                                 yellowishwhite banding can be seen; myocardium is dull,
C. Interstitial proliferative                                    darkyellow. What process caused this pathology?
D. Serous focal                                                  A. Fatty vascularstromal degeneration
E. Serous diffuse                                                B. Fatty parenchymatous degeneration
                                                                 C. Carbohydrate degeneration
9. Colonoscopy of a patient ill with dysentery revealed          D. Hyaline degeneration
that mucous membrane of his large intestine is hyperemic,        E. Amyloidosis
edematic, its surface was covered with greyandgreen
coats. Name the morphological form of dysenteric collitis:       18. A 48 y.o. patient was admitted to the hospital with
A. Fibrinous                                                     complaints about weakness, irritability, sleep disturbance.
B. Necrotic                                                      Objectively: skin and scleras are yellow. In blood:
C. Purulent                                                      conjugated bilirubin, cholalemia. Feces are acholic. Urine
D. Ulcerous                                                      is of dark colour (bilirubin). What jaundice is it?
E. Catarrhal                                                     A. Gilbert's syndrome
                                                                 B. Parenchymatous
10. A woman has been applying a new cosmetic                     C. Mechanic
preparation for a week that resulted in eyelid inflammation      D. Hemolytic
accompanyed by hyperemia, infiltration and painfulness.          E. CriglerNajjar syndrome
What type of allergic reaction was developed?
A. I                                                             21. A 56 yo. patient has been suffering from thyreotoxicosis
B. IV                                                            for a long time. What type of hypoxia can be developed?
C. С III                                                         A. Tissue
B.   Mixed                                                    B.   Pancreas
C.   Circulatory                                              C.   Thyroid
D.   Hemic                                                    D.   Thymus
E.   Respiratory                                              E.   Liver

22. Microscopical examination of a removed appendix           32. Autopsy of a 58 y.o. man revealed that bicuspid valve
revealed an edema, diffuse neutrophilic infiltration of       was deformed, thickened and unclosed. Microscopically:
appendix wall along with necrosis and defect of mucous        foci of collagen fibrilla are eosinophilic, react positively
membrane with affection of its muscle plate. What             to fibrin. The most probably it is:
appendicitis form was developed?                              A. Fibrinous inflammation
A. Phlegmonous                                                B. Hyalinosis
B. Apostematous                                               C. Amyloidosis
C. Superficial                                                D. Fibrinoid swelling
D. Ulcerophlegmonous                                          E. Mucoid swelling
E. Gangrenous
                                                              34. A patient with clinical presentations of
23. A patient is ill with diabetes mellitus that is           immunodeficiency       went     through     immunological
accompanied by hyperglycemia of over 7,2 millimole/l on       examinations. They revealed significant loss of cells that
an empty stomach. The level of what blood plasma protein      form rosettes with erythrocytes of a ram. What conclusion
allows to estimate the glycemia rate retrospectively (48      can be made according to the analysis data?
weeks before examination)?                                    A. Decrease of natural killer cell rate
A. Glycated hemoglobin                                        B. Insufficiency of effector cells of humoral immunity
B. Ceruloplasmin                                              C. Decrease of T-lymphocytes rate
C. Creactive protein                                          D. Decrease of complement system rate
D. Fibrinogen                                                 E. Decrease of B-lymphocytes rate
E. Albumin
                                                              35. Autopsy of a patient who suffered from croupous
24. A 45 y.o. patient consulted a doctor about                pneumonia and died from pneumococcal sepsis revealed
plaqueshaped formation on his neck. Histological              900 ml of turbid greenishyellow liquid in the right pleural
examination of biopsy skin material revealed tumourous        cavity. Pleural leaves are dull, plephoric. Name the
cells of round and oval form with thin ring of basophilic     clinicopathological form of inflammation in the pleural
cytoplasma that resemble of cells of basal epidermal layer.   cavity
What tumour is it?                                            A. Phlegmon
A. Basalioma                                                  B. Chronic abscess
B. Syringoadenoma                                             C. Empyema
C. Epidermal cancer                                           D. Fibrinous inflammation
D. Hydradenoma                                                E. Acute abscess
E. Trichoepithelioma
                                                              36. Mucous membrane of the right palatine tonsil has a
29. An experimental animal was first sensibilized             painless ulcer with smooth lacquer fundus and regular
whereupon an antigen dose was introduced subcutaneously.      cartilaginous edges. Microscopically: inflammatory
This injection resulted in the development of a fibrinous     infiltration that consists of lymphocytes, plasmocytes, a
inflammation with alteration of vessel walls, basal           small number of neutrophils and epithelioid cells;
substance and fibrous structures of connective tissue in      endovasculitis and perivasculitis. What disease is it?
form of mucoid and fibrinoid swelling and necrosis. What      A. Pharyngeal diphtheria
immunological reaction took place?                            B. Ulcerous necrotic Vincent's angina
A. Delayed type hypersensitivity                              C. Actinomycosis
B. Immediate hypersensitivity                                 D. Syphilis
C. Reaction of transplantation immunity                       E. Tuberculosis
D. Normergic reaction
E. Granulomatosis                                             37. A 27yearsold woman has been suffered by diabetes
                                                              mellitus since she had been a child. During the last years
30. Examination of cell culture got from a patient with       the arterial hypertension and proteinuria have occurred. She
lysosomal pathology revealed accumulation of great            has died with signs of uremia. What typical changes were
quantity of lipids in the lysosomes. What of the following    found out in autopsy?
diseases is this disturbance typical for?                     A. “Cor villosum”
A. Wilson disease                                             B. “Tiger heart”
B. Galactosemia                                               C. “Armor heart”
C. TaySachs disease                                           D. Mesenchymal fatty dystrophy of the heart
D. Phenylketonuria                                            E. Purulent pericarditis
E. Gout
                                                              38. A 43yearsold patient has a burn of right hand. The
31. A teenager was irradiated with high radiation dose that   exfoliation of epidermis and formation of bubbles filled by
resulted in serious damages of lymphoid system, lysis of      semitransparent fluid is manifested in the palm and the back
many lymphocytes. Restoration of normal hemogram is           surface of the hand. What kind of inflammation occurs in
possible due to the functioning of the following gland:       that case?
A. Adrenal                                                     A. A.Serous
B.   B.Purulent                                                   E. After anemia
C.   C.Fibrinous
D.   D.Putrefactive                                               45. A 23yearsold patient got a lesion of the liver because of
E.   E.Catarrhal                                                  trauma. In time a structure and functions of that organ was
                                                                  restored completely. What kind of regeneration did occur in
39. A patient was admitted in the hospital with diagnosis of      that case?
trichinellosis. What kind of inflammation develops within         A. Restitution
the muscles around the parasites?                                 B. Pathologic regeneration
A. Proliferative granulematous                                    C. Physiologic regeneration
B. Exudative fibrinous                                            D. Metaplasia
C. Exudative purulent                                             E. Substitution
D. Exudative serous
E. Proliferative interstitial                                     46. A 4yearsold girl was operated because of an acute
                                                                  appendicitis. During the operation in the retroperitoneal
40. A 22yearsold patient was admitted in hospital with            space it was found that the right kidney is less by 1/3 in
complaints on heavy nasal breathing. During the                   comparison with the left one. The diameter of the right
examination of nasal cavity it was found the thickened            renal artery was 0,3 cm, the left one was 0,4 cm. What
mucous membrane, a lot of mucus and nodular infiltrates           pathologic process did occur in that case?
without erosions. It was diagnosed as rhinoscleroma of            A. Hypoplasia
nose. It was took the biopsy. What typical morphological          B. Pathologic atrophy
changes may be found?                                             C. Physiologic atrophy
A. Granulomas with Mikulicz’s cells                               D. Aplasia
B. Granulomas with Virchow’s cells                                E. Agenesia
C. Granulomas with Langhan’s cells
D. Granulomas with foreign body cells                             47. A patient has suffered with mitral insufficiency that
E. Interstitial inflammation                                      complicated by chronic cardiac insufficiency with grossly
                                                                  marked edema of the lower extremities. Suddenly cyanosis
41. During the histological examination of the fragment of        of the face is manifested with attempt standing up and he
mucous membrane of urine bladder of the patient suffered          has died. What the most probably complication developed
with chronic cystitis it was found out: the transitional          in patient?
epithelium focally is replaced by stratified nonkeratinized       A. Thromboembolism of the pulmonary artery
epithelium. These changes can manifest development of:            B. Venousarterial reaction
A. Metaplasia                                                     C. Infarction of myocardium
B. Dysplasia                                                      D. Chronic venous hyperemia
C. Leukoplakia                                                    E. Hypertension stroke
D. Hyperkeratosis
E. Hyperplasia                                                    48. A child fell down the tree and got a simple fracture of
                                                                  the one of hand’s bones. In a time healing occurred. Call the
42. A 1/3 of the liver with a multicellular echinococcus was      kind of regeneration and cells, which taking part in
resected during the operation. What typical changes of the        restoration.
liver tissue surrounding the parasites were found out by the      A. Reparative regeneration, osteoclasts and osteoblasts
pathologist during the histological examination of the            B. Physiological regeneration, octeoclasts
resected part of the organ?                                       C. Pathological regeneration, osteoclasts
A. Formation of nonspecific granulomas                            D. Intracellular regeneration, osteocytes
B. Flabby consistence of the liver                                E. Cellular regeneration, osteoclasts and osteoclasts
C. Formation of specific granulomas
D. Exudative tissue reaction                                      49. A pilot has died because of depressurization of the cabin
E. Hydropic dystrophy                                             of the airplane. In autopsy it was found out a gross marked
                                                                  rigor mortis, emphysema of subcutaneous tissue of the body
43. During the autopsy it was found the enlarged firm liver       and face. There were also edema and perivascular
with rounded margins; the surface of the cut looks                hemorrhages within the lungs. There was a foamy and fluid
nutmeglike of yellowbrown colour with darkred spots.              blood within the veins. The histological examination was
What pathologic process do underlie of those changes?             showed a lot of bubbles within the vessels of all inner
A. Chronic venous hyperemia                                       organs, a fat dystrophy of the liver and a quantity of small
B. Arterial hyperemia                                             gray softening areas in the brain. Call the more probable
C. Arterial ischemia                                              cause of those changes.
D. Fat embolism                                                   A. Gas embolism
E. Chronic bleeding                                               B. Air embolism
                                                                  C. Tissue embolism
44. There is a redness of skin above the site of obturation, if   D. Thromboembolism
the thrombus obstructed the main artery of upper extremity.       E. Fat embolism
Call the kind of arterial hyperemia appearing as a result of
disturbance of bloodstream.                                       50. A man has died in 8th day since beginning of the
A. Collateral                                                     disease. It was diagnosed dysentery. During the autopsy it
B. Angioneurotic                                                  was found out a thickened wall of the sigma and rectum,
C. Inflammatory                                                   fibrinous membrane on the surface of mucous membrane.
D. Physiological                                                  Histologically: there is a deep necrosis of mucous
membrane with infiltration of necrotic masses with fibrin.        C. Hyperplasia
What kind of colitis does correspond to those changes?            D. Neoplasm of kidney
A. Diphtheritic                                                   E. Pyelonephritis
B. Catarrhal
C. Ulcerative                                                     57. A woman suffered with dysfunctional metrorrhagia was
D. Chronic                                                        made a diagnostic abortion. Histologically in the scrape
E. Gangrenous                                                     there were a lot of small stamped glandulas covered by
                                                                  multirowed epithelium. The lumens of some glandulas were
51. A patient suffered with rheumatic fever with a mitral         cystically extended. Call the variant of general pathologic
valve disease of the heart for a long time. He has died as a      process in the endometrium.
result of cardiacpulmonary insufficiency. During the              A. Glandularcystic hyperplasia of endometrium
autopsy it was found out a brown induration of the lungs.         B. Atrophy of endometrium
Call kind of hemodynamic disorders causing those changes.         C. Metaplasia of endometrium
A. Chronic left ventricle failure                                 D. Neoplasm of endometrium
B. Chronic right ventricle failure                                E. Hypertrophic growth
C. Acute left ventricle failure
D. Acute right ventricle failure                                  58. A 13yearsold child was admitted into the hospital with
E. Portal hypertension                                            complaints of fever, weakness, and intense pain in the right
                                                                  thigh. From anamnesis it was defined that he has bitten by
52. A patient suffered by stenosis of mitral valve of 4th stage   dog 3 days ago. During the visual examination it was found
has died as a result of cardiacpulmonary insufficiency.           out redness, swelling and acute painfulness along right
Autopsy was showed a hemosiderosis of the lungs. Call the         thigh. After excision it was made the histological
kind of hemodynamic disorder in the lungs.                        examination, which was showed a considerable
A. Local venous hyperemia                                         accumulations of polymorphonucleus leukocytes between
B. General venous hyperemia                                       the mussel fibers partly undergone by purulent fusion.
C. Arterial hyperemia                                             Diagnose that condition.
D. Inflammatory hyperemia                                         A. Phlegmon
E. Hemorrhage                                                     B. Abscess
                                                                  C. Gangrene
53. A patient has died as a result of cardiac insufficiency. In   D. Empyema
anamnesis he had got a pulmonectomy in account of cyst of         E. Croupous inflammation
the right lung. During the autopsy enlarged left one was
found out. Call the pathologic process in the left lung.          59. A patient with severe fracture of both femoral bones has
A. Replacement hypertrophy                                        died in a 4 hours after trauma. What additional staining of
B. Neurohumoral hypertrophy                                       microscopic sections is it necessary to make for exacter
C. Dyscirculatory atrophy                                         diagnosis of the cause of death?
D. Dysfunctional atrophy                                          A. The lungs and brain by Sudan III
E. Physiologic hypertrophy                                        B. The lungs and kidney by Congored
                                                                  C. The lungs and liver by Perl’s reaction
54. A patient has undergone to amputation of lower                D. The brain and kidney by Congored
extremity. In a time a painful nodules appeared in a stump.       E. The kidney and heart by toluidinblue
Amputatious neuromas were found out during the
microscopical examination. What kind of pathological              61. A 59yearsold woman was admitted into the hospital
processes does those formations relate to?                        with signs of exicosis being in deep shock. After her death
A. Regeneration                                                   it was made autopsy showed presence of vibrio cholerae,
B. Dystrophy                                                      moderate mononuclear infiltrations in the mucosal lamina
C. Inflammation                                                   propria of intestine. What kind of shock was the cause of
D. Hyperemia                                                      death of the patient?
E. Metaplasia                                                     A. Hypovolemia
                                                                  B. Cardiogenic
55. After deep burns of the skin a patient has got a keloid       C. Anaphylactic
scarring. What kind of pathologic processes does those            D. Traumatic
formations relate to?                                             E. Toxic
A. Incomplete regeneration (substitution)
B. Complete regeneration (restitution)                            62. A cattle worker has died with signs of severe
C. Atrophy                                                        intoxication in a 2nd day after beginning of disease. Autopsy
D. Hypertrophy                                                    was showed the enlarged flabby spleen, on the surface of
E. Metaplasia                                                     the cut of darkcherry coloured; the scrape of the pulp is
                                                                  abundant. The leptomeninges are swelling, infiltrating by
56. A patient suffered with urolithiasis has died as a result     blood of darkred colour (“a red cap”). It was defined a
uremia. Autopsy is showed an enlargement of right kidney,         microorganism – bacillus anthracis – and diagnosed
its parenchyma has been thinned; the pelvis and calices           anthrax. What kind of inflammation does occur in that
have been extended, filled with fluid. In the orifice of ureter   disease?
there has been a calculus. Call the pathologic process in the     A. Hemorrhagic
right kidney.                                                     B. Phlegmonous
A. Hydronephrosis                                                 C. Fibrinous
B. Replacement hypertrophy                                        D. Purulent
E. Putrefactive                                                 D. Tuberculosis.
                                                                E. Actinomycosis.
63. A 65yearsold patient suffered by thrombophlebitis of
the deep veins of both legs has died suddenly. Autopsy was      69. The patient with severe intoxication has died. During
showed free lying dry friable red masses with a dull            the autopsy it was found out: the left tonsil was enlarged,
crimped surface within the truncus pulmonalis and               firm; in section it contained a lot of cavities filled by
bifurcation of the lung artery. What process within the         yellow–greenish masses. The tissues of the neck, and
vessels did pathologist find?                                   mediastinum were undergoing with suppurative fusion.
A. Thromboembolism                                              Microscopically: there were a lot of abscesses, in the center
B. Thrombosis                                                   of which intensive basophilic formations were defined.
C. Tissue embolism                                              Those formations consist of short sticklike elements,
D. Postmortem clot                                              surrounded with radiating projections (“ray fungus”). Such
E. Hemangioma                                                   changes characterized:
                                                                A. Actinomycosis.
64. A patient was admitted into the hospital with complaints    B. Amebiasis.
to feel sick, pains in the epigastria after meal. During the    C. Brucellosis.
gastroscopy it was found out hyperemia, edema of mucous         D. Tuberculosis.
membrane of the stomach, an excessive amount of viscous         E. Scarlet fever.
grayish mucus. What kind of inflammation did develop in
the stomach?                                                    70. A 34yearsold woman has got pain, redness and swelling
A. Catarrhal                                                    of finger on account of careless use of iron. The blister
B. Fibrinous                                                    filled by transparent yellowish fluid appears in some
C. Serous                                                       minutes. What pathologic process do such signs
D. Hemorrhagic                                                  characterize?
E. Purulent                                                     A. Exudative inflammation.
                                                                B. Traumatic edema.
65. During the autopsy it was found out: the lungs are firm,    C. Hypertrophy.
have brown colour because of accumulation of endogenous         D. Proliferative inflammation.
pigment. It’s known that the patient has suffered with          E. Hydropic dystrophy.
venous congestion in the pulmonary circulation. What
process did cause such changes?                                 71. The patient with chronic glomerulonephritis has died
A. Hemosiderosis                                                with accompanied signs of uremia. The cadaver had
B. Calcinosis                                                   specific uremic smell. Macroscopically: there were a lot of
C. Jaundice                                                     pilelike whitishgrayish puttings on the surface of
D. Melanosis                                                    pericardium. The vessels filled with blood became visible
E. Porphiria                                                    after removal of such formations. What process did develop
                                                                in the pericardium?
66. During the autopsy it was found out the cavity 2,5x1,5      A. Fibrinous inflammation.
sm within the right frontal part of the brain. It was filled    B. Organization.
with transparent fluid and its walls were smooth and of         C. Proliferative inflammation.
brownish colour. What process did develop in the brain?         D. Hemorrhagic inflammation.
A. Cyst as an outcome of hemorrhage.                            E. Arterial hyperemia.
B. Grey softening of brain.
C. Abscess of brain.                                            72. The patient complains of local pain in the back of the
D. Birth defect of brain.                                       head, increasing of the temperature in that area.
E. Cyst as a outcome of gray softening                          Macroscopically: there is a conical infiltrate of purpleblue
                                                                colour with yellowgreenish apex, which protrudes under the
67. During the histological examination of the fragment of      surface of the skin. Diagnose it:
cervix it was found out the area of inflammatory infiltration   A. Furuncle.
with involvement of walls of small vessels. The infiltrate      B. Phlegmona.
was composed of plasma cells, lymphocytes, epithelioid          C. Abscess.
cells and areas of sclerosis and hyalinosis. What disease       D. Fistula.
does such picture characterize?                                 E. Empyema.
A. Syphilis.
B. Tuberculosis.                                                73. During the histological examination of the biopsy the
C. Leukoplakia.                                                 pathologist has found out granulomas within the livers.
D. Erosion of cervix.                                           They contain mainly Tlymphocytes and epithelioid cells,
E. Condyloma.                                                   and solitary giant Langhan’s cells. In the center of
                                                                granulomas there was a small area of caseous necrosis.
68 During the histological examination of the biopsy it was     What pathologic process is characterized by those changes?
found out the granulomas composing of lymphocytes,              A. Proliferative inflammation.
epithelioid cells, plasma cells, macrophages with foamy         B. Coagulative necrosis.
pale cytoplasm (Mikulicz’s cells), a lot of hyaline balls.      C. Liquefactive necrosis.
What disease do you think about?                                D. Neoplasm.
A. Rhinoscleroma.                                               E. Exudative inflammation.
B. Leprosy.
C. Syphilis.
74. A 5yearsold girl suffered from diphtheria. She has died   E. Defect of telomere Хchromosoma
in three days because of asphyxia caused by diphtheria’s
croup. During the autopsy it was found out, that mucous       83. During the autopsy the pathologist has marked: the
membranes of larynx, trachea and bronchi were thickening,     expressed linear shrinkage and thickening of extremities
swelling, dull, covering by grayish membranes, which          with formation on a skin of large tucks. The head is
easily came off. What process did develop in the larynx?      increased, nose is saddle, the oral cavity is semiopen,
A. Croupous inflammation.                                     tongue is thick, neck is short, bodies of spondyles are
B. Serous inflammation                                        dwarfed, hypoplasia of thorax is combined with hypoplasia
C. Diphteritic inflammation                                   of lungs. Which inherent malformation of a musculoskeletal
D. Proliferative inflammation                                 system is it characteristic for?
E. Suppurative inflammation                                   A. Lethal micromelia
                                                              B. Chondroplasia
75. As a result of a microscopic examination of the           C. Imperfect bone formation
fragment of the skin the granulomas were found out; they      D. Inherent marble bones
composed of a lot of macrophages with a few amount of         E. Inherent Oppenheim myatonia
lymphocytes and plasma cells. Besides large macrophages
with fat vacuoles occurred, the microorganisms were           84. In the autopsy of 48yearold man [miner which got
packed in those vacuoles as “cigars in the box” (Virchow’s    hematite] the pathologist has found out increased brownred
cells). Granulation tissue had a good vascularization. What   dense lungs. Microscopic picture is: a moderate
disease are such granulomas characterized for?                pneumosclerosis, submiliary and miliary nodules, which
A. Leprosy                                                    consist of dust cells with spots (positive reaction to iron). In
B. Tuberculosis.                                              lymph nodes there are a lot of dust and considerable diffuse
C. Syphilis.                                                  sclerosis. Which professional disease is it?
D. Rhinoscleroma.                                             A. Red siderosis
E. Actinomycosis.                                             B. Black siderosis
                                                              C. Aluminosis
76. During the microscopic examination of bioptic fragment    D. Berilliosis
of the skin the granulomas were found out containing          E. Anthracosis
epithelioid cells, surrounded with Tlymphocytes. Between
the epithelioid cells the solitary giant polynuclear          85. In the autopsy of a body of the miner, which got
Langhan’s cells located. There were areas of caseous          hematite, the pathologist has found out: black lungs, similar
necrosis in the center of some granulomas. Blood vessels      on lungs at an anthracosis. Microscopically: moderate
were absent. What disease do such changes characterize?       pneumosclerosis, submiliar and miliar nodules, which
A. Tuberculosis.                                              consist of dust cells with spots [positive reacton to iron]. In
B. Syphilis.                                                  lymph nodes there are a lot of dust and considerable diffuse
C. Leprosy.                                                   sclerosis. Your diagnosis?
D. Rhinoscleroma.                                             A. Black siderosis
E. Hodgkin’s disease.                                         B. Red siderosis
                                                              C. Aluminosis
77. In a biopsy of a сervix uteri of a26yearold women the     D. Berilliosis
diagnosis following was established: pseudoerosion. What      E. Caplan syndrome
microscopical changes has the pathologist revealed?
A. Local changes of a stratified squamous epithelium          90. A 45yearold man has died because                    of
     on single-layer prismatic one                            pulmonarycardiac insufficiency. In autopsy the pathologist
B. Cellatypia of an epithelium of an mucosal epithelium       has found out croupous pneumonia in lower lobe of left
C. Keratinization of an epithelium of an           mucosal    lung, 350ml of greenishyellow fluid in the left pleural
     epithelium                                               cavity. Microscopically it contained many neutrophils. Call
D. “Carcinomatous pearls”                                     the complication of pneumonia
E. Local inflammation and necrosis in mucosa                  A. Empyema of pleura
                                                              B. Fibrinous pleuritis
80. Inheritable predilection to development of lung           C. Pneumothorax
emphysema is connected with genetic disturbance, which        D. Hydrothorax
one shows in decreasing production of:                        E. Hemothorax
A. Alpha1antitrypsin
B. Creactive protein                                          98. A 60yearold miner has died because of chronic
C. Antistreptolysin 0                                         pulmonarycardiac insufficiency. In autopsy the pathologist
D. Hyaluronidasa                                              has found out: areas of dystelectasis, induration,
E. Complement                                                 pneumosclerosis in the both lungs, local emphysema in the
                                                              apex. Cut surface was slatyblack. Diagnose this disease.
82. Gangliosidlipidosis is characterized by deficiency of     A. Anthracosis
enzyme alpha –hexoaminidaze and accumulation of               B. Silicosis
gangliosides in lysosomas of nervous cells.It is connected    C. Talcosis.
with genetical defect, which is determined as:                D. Asbestosis
A. Defect of one gene                                         E. Aluminosis.
B. Translocation of a gene
C. Defects of several genes                                   108. In 56 year –old patient has suffered from
D. Trisomy of 21 chromosoma                                   bronchoectatic disease and hemoptysis, the edema of face
and waist have appeared. The protein (33 mg/l) was found       A.   Sequestrum
in urine. Pulmonary hemorrhage was the cause of patient’s      B.   Dry gangrene
death. In autopsy: enlargement of kidneys was found; the       C.   Wet gangrene
kidneys were densed with lardaceus surface of section.         D.   Infarction
Histologically: the deposition of homogenous eosinophilic      E.   Caseous necrosis
masses colored with Congo red and given of metachromasia
with methyl violet color in glomeruli and canals were          117. The 46year old woman has suffered from rheumatic
found. What pathological process took place in the patient?    fever with combined mitral heart disease. In autopsy the
1. Amyloidosis                                                 leaflets of mitral valve are thicken, intertwisted and have
B. Grainish degeneration;                                      stony dense. What pathological process has led to these
C. Fatty degeneration;                                         changes?
D. Mucoid degeneration;                                        A. Amyloidosis;
E. ;Hyalinosis;                                                B. Hyaline changes;
                                                               C. Dystrophyc calcification;
111. In 45year old patient died from sudden cardiac death      D. Metabolic calcification;
the symmetrical type of adipose heart of third degree; the     E. Metastatic calcification;
rupture of right ventricle’s wall with hemopericardium and
redundant accumulation of fat under epicardium were found      118. Patient is suffered from cholera, which is during with
in autopsy. Microscopically: the adipose tissue grows from     dehydratating, cyanosis and convulsions. In result of
epicardium into myocardium with atrophy of fibers of           massive infusion therapy the exicosis is diminished, but
muscle. What process is more probable?                         anuria is remained. Patient is dead of uremia. Choose
A. Adipose heart;                                              correct position for this case.
B. Hypertensive disease;                                       A. Necrotic nephrosis with cortical necrosis takes place
C. Ischemic heart disease;                                         in the kidneys,
D. Fatty degeneration of myocardium;                           B. Choleral typhoid is developed,
E. Acute myocardial infarction;                                C. Development of uremia is connected with acute
                                                                   glomerulonephritis,
112. The 48yearold patient, suffered from fibrouscavernous     D. Fibrinouse colitis is found in autopsy,
tuberculosis, has complained of weakness, reduction of         E. Exicosis is due to action of virus exotoxin.
daily secretion of urine, edema of the body and extremities
and increasing of blood pressure to 180/90. The increasing     119. Please, choose incorrect statement.
of protein, presence of hyaline and grain cylinders and        A. Degeneration is characterized by disorder of tissue
erythrocytes were found under analysis of urine. The patient       nutrition and in every time lead to distraction of
has died in a month because of the insufficiency of kidney.        nucleus and whole cell
In autopsy, the enlargement of the heart and "lardaceus"       B. There are extracellular and intracellular accumulations
kidneys with mass of more than 240 g were found. What is       C. Pathological calcification may be in two forms:
the complication of fibrosecavernouse tuberculosis?                dystrophic and metastatic calcification
A. Аmyloidosis;                                                D. Degenerations may be caused by hereditary factors
B. Glomerulonephritis;                                         E. Degenerations can appear in one organ or in the whole
C. Nephrotic syndrome;                                             body
D. Pyelonephritis;
E. Nephrosclerosis;                                            120. In autopsy: enlargement of kidneys was found; the
                                                               kidneys were densed with lardaceus surface of section.
114. In patient with jaundice the following data were          Histologically: the deposition of homogenous eosinophilic
established: in serum the increasing of bilirubin because of   masses colored with Congo red and given of metachromasia
the unconjugated form; in faeces and urine increasing of       with methyl violet color in glomeruli and canals were
stercobilin; the level of conjugated (direct) bilirubin in     found. What pathological process took place in the patient?
serum is normal. What type of jaundice takes place?            A. Amyloidosis
A. Haemolytic jaundice;                                        B. Grainish degeneration;
B. Jaundice of newborns;                                       C. Fatty degeneration;
C. Parenchymatous (hepatic) jaundice;                          D. Mucoid degeneration;
D. Gilbert’s disease;                                          E. ;Hyalinosis;
E. Mechanical (posthepatic) jaundice;
                                                               121. Choose one incorrect statement:
115. In autopsy of the 83yearold patient’s body, has died      A. The mechanism of decomposition is a base of
because of stomach cancer, the heart and the liver were            genetic storage diseases;
diminished in size, condensed, with brown color. What          B. Decomposition (phanerosis) – disintegration of
pathological process took place in these organs?                   membranous structures of cells and intercellular
A. Sclerosis;                                                      matrix;
B. Mesemchymal degeneration;                                   C. Infiltration – redundant accumulation (deposition) of
C. Parenchymal degeneration;                                       metabolites into the cells and intercellular matrix;
D. Necrosis;                                                   D. Perverted synthesis – synthesis of abnormal substances
E. Brown atrophy                                                   in the cells and tissues;
                                                               E. Transformation – formation of one type of metabolism
116. Fragment of dead tissue, which can’t be autolized,            products from common initial substances for protein,
replaced by connective tissue and which is localized among         fats and carbohydrates.
alive tissue is named…
122. Intracellular accumulation of fat in liver is             lowerlobar pneumonia with expectoration of mucus with
characterized by every statements, except one:                 pus. In autopsy in 910 segments of the right lung the cavity
A. Fat is accumulated in extracellular spaces                  with dense walls filled with purulent masses, was found.
B. There are small fat vacuoles in cytoplasm around the        The whitish path comes from the cavity toward the radix of
    nuclei of hepatocytes                                      the lung. Microscopically in was established that the cavity
C. Liver enlarged and becomes yellow                           is divided from saved lung tissue with thin membrane,
D. Its consistency is soft and greasy                          which consists of two layers: internalgranulation tissue, and
E. Fat is stained by special stain Sudan 3, and becomes        external –connective tissue. What diagnosis is more
    orangered color                                            probable?
                                                               A. Chronic abscess;
123. A 55 year old man has died after chronic                  B. Pulmonary gangrene;
glomerulonephritis, chronic renal insufficiency. In the        C. Acute pulmonary abscess;
autopsy the pathologist has found out characteristic changes   D. Chronic pneumonia;
in kidneys for this disease, also fibrinous pericarditis,      E. Bronchoectatic disease
pleuritis, bronchitis. Call the cause of the fibrinous
inflammation in serosal and mucosal layers.                    150. Cyanosis, enlargement of the liver, edema of the low
A. Uremia.                                                     extremities as a result of insufficiency of the right ventricle
B. Hypolipidemia                                               of the heart were found in a patient. What is the cause of the
C. Hyperlipidemia                                              development of such insufficiency?
D. Arterial hypertension                                       A. Hypertension of the small circle of the blood
E. Arterial plethora                                                circulation
                                                               B. Functional shunts in lungs
140. During autopsy of the patient died from chronic           C. Increasing of the venous pressure
cardiac insufficiency the enlarged dense rusty colored lungs   D. Hypercathecholemia
with growth of gray color connective tissue around bronchi     E. Cardiogenic cirrhosis of the liver
and vessels were found. How can we call this process in
lungs?                                                         153. In 53 yearold patient, who has suffered by
А        Brown induration of the lung                          bronchoectatic disease and hemoptysis, the edema of face
В        Hemorrhagic pneumonia                                 and waist have appeared. The protein (33 mg/l) was found
С        Interstitial emphysema                                in urine. Pulmonary hemorrhage was the cause of patient’s
D        Chronic obstructive emphysema                         death. In autopsy: enlargement of kidneys was found; the
E         Primary idiopathic emphysema;                        kidneys were dense with lardaceous surface of section.
                                                               Histologically: the deposition of homogenous eosinophilic
143. In 77yearold patient suffered with atherosclerosis the    masses colored with Congo red and given of metachromasia
pain has appeared in the right foot. The foot is enlarged in   with methylviolet color in glomeruli and canals were found.
size, its skin has black color and is macerated; the           What pathological process took place in the patient?
demarcation line is not clear. What pathological process       A. Amyloidosis;
takes place in a patient?                                      B. Fatty degeneration;
A. Wet gangrene                                                C. Mucoid degeneration;
B. Coagulative necrosis                                        D. Grainish degeneration
C. Sequestrum                                                  E. Hyalinosis
D. Dry gangrene
E. Noma                                                        155. In the case of rheumatic heart disease, pericarditis is
                                                               characterized by:
145. A 65 year old patient, who suffered from                  A. “Shaggy heart” (“cor villosum”)
arteriosclerosis, has been hospitalized in surgical            B. “Cor pulmonale”
department because he had purulent peritonitis. Thrombosis     C. “Tiger heart”
of mesenteric arteries was found during operation. What is     D. Fatty change
the most probable cause of peritonitis?                        E. “Bull heart”
A. Hemorrhagic infarction
B. Angiospastic ischemia                                       156. In a 45yearold patient the ulcerativenecrotic damage of
C. Angioneurotic edema                                         the mucosa of the oral cavity takes place; also the spread
D. Stasis                                                      lymphadenopathy, slight spleno and hepatomegaly, diffuse
E. Chronic congestion                                          hemorrhages in the skin and mucous membranes were
                                                               found out. In blood analysis the increasing of leukocytes (to
148. Under microscopic investigation the postinfarction        100 10 per 1 ml) at the account of lymphoblasts takes place.
cardiosclerosis has been found out. Around cardiosclerotic     What diagnosis is more probable?
area myocardiocytes were enlarged in size and had large        A. Acute lymphoblastic leukemia
hyperchromic nuclei riched in DNA. What process taking         B. Chronic myelocytic leukemia
place in myocardiocytes is more probable?                      C. Chronic lymphocytic leukemia
A. Regenerative hypertrophy                                    D. Plasmocytosis
B. Physiologic regeneration                                    E. Acute promyelocytic leukemia;
C. Complete reparative regeneration
D. Pathologic regeneration                                     159. Chronic venous congestion of organs, hypertrophy of
E. Hypertrophy because of increased workload                   the left heart ventricle with cardiosclerosis, volumetrical
                                                               white-yellowish plaques in intima of aorta with
149. 56yearold patient has suffered from rightside             atheromatous masses in the center, which are deeper in the
wall, are found in the result of autopsy of 70year man, who       E. Perineural
died of heart insufficiency and who suffered from angina
pectoris, hypercholesterinemia and obesity. Which                 170. At the autopsy of 60 yearold male patient, in the
pathological process is more possible in aorta?                   myocardium of the front wall of the left ventricle of the
A. Atheromatosis                                                  heart a gray dense focus of 5 х 4 cm, of irregular shape with
B. Lipoidosis                                                     clear borders, fibrous structure has been found out. What
C. Arteriolosclerosis                                             pathological process in the myocardium has revealed the
D. Liposclerosis                                                  pathologist?
E. Calcification                                                  A. Postinfarctional cardiosclerosis
                                                                  B. Diffuse smallfocal myocardiosclerosis
161. All statements are correct in the relations of hyalinosis,   C. Myocarditis
except one:                                                       D. Petrification of the myocardium
A. The hemorrhage and hemoliysis play important role              E. Infarction
    in the hyalinosis formation
B. Hyaline is accumulated in the connective tissue and            171. In 40yearold patient, the tumor, which grew under skin
    looks like cartilage                                          of spine was resected. The histologic diagnosis: a lipoma.
C. Mainly arterioles and arteries undergo to hyalinosis           What principle of the tumors’ classification did the
D. Hyalinosis is rare reversible process                          pathologist use when created his conclusion?
E. There are not macroscopical changes of the organs              A. Gistogenesis
    usually, but hyalinosis may lead to deformation and           B. Of biochemical features
    wrinkle of the organs                                         C. Of ultrastructural features
                                                                  D. Of physicochemical features
166. In 60yearold patient, during examination the cancer of       E. Macrostructure of an organ
a prostate gland with metastases in the lower parts of the
column, pelvic bones and pathological fracture of a femur         172. At histologic research of a biopsy specimen from an
was found out. What pathological process is a basis of the        auricle of a heart of a patient with rheumatic disease the
development of metastases?                                        foci of a mucoid swelling, fibrinoid necrosis of a connective
A. Tissue embolism                                                tissue has been found out. What immune response has
B. Bacterial embolism                                             developed in tissues of the auricle of the heart?
C. Airembolism                                                    A. Hypersensitivity of an immediate type
D. Embolism by foreign matters                                    B. Hypersensitivity of a delayed type
E. Necrosis of tissue                                             C. Reaction of the transplantative immunity
                                                                  D. Normergic reaction
167. In patient with chronic cystitis during the investigation    E. Exudative reaction
of biopsy of a mucosa of urinary bladder the foci of a
squamous nonkeratinized epithelium are found out. It can          178. Call a kind of an infarct according to macroscopic
be a sign of development of:                                      signs, which is characteristic in myocardium.
A. Metaplasia                                                     A. White with a hemorrhagic halo
B. Leukoplakia                                                    B. Hemorrhagic
C. Hyperplasia                                                    C. Anemic
D. Dysplasia                                                      D. Mixed
E. Hyperkeratosis                                                 E. Red

168. During the laparotomy in 49 yearold male patient, the        179. What type of exudate appears in pericardium cavity at
tumor has been found out in the field of a sigma with             a rheumatic pericarditis?
growth through all its layers and an occlusion of the lumen       A. Serous-fibrinous
of an intestine. The biopsy has been taken and colonostoma        B. Putrefactive
has been overlapped. The clinical diagnosis after operation:      C. Hemorrhagic
a cancer of sigma. What kind of tumor is growth in relation       D. Purulent
to tissues?                                                       E. Catarrhal
A. Infiltrative
B. Expansive                                                      180. At autopsy of a patient died because of a cerebral
C. Endophytic                                                     hemorrhage, strongly enlarged dense and anemic kidneys
D. Exophytic                                                      ( size :6х3х2см weight         60.0,) with a uniformly
E. Multicentric                                                   smallgranulated surface and with uniformly thinned cortex
                                                                  on a cutsurface have been found out. The changes in kidney
169. In 36 yearold patient a dark brown color patch on skin       are a sign of:
of a dextral foot has been resected. The histologic research      A. Of arteriosclerotic nephrosclerosis
has not been carried out. In 5 months in dextral inguinal         B. Of atherosclerotic nephrosclerosis
area a conglomeration of lymph nodes has appeared. At             C. Secondary shrinkage of kidney
histologic research of one of them the metastasis of              D. Gouty (podagric) kidneys
melanoma has been found out. What pathway of spreading            E. Amiloidshrinkaged kidneys (“lardaceous kidneys”)
of tumor takes place?
A. Limphogenous spread                                            183. A 50yearold man had rheumatic mitral disease. He has
B. Hematogenous spread                                            arrived in clinic with sings of active rheumatism. On a
C. Anaplastic                                                     background of increasing cardiovascular insufficiency has
D. Mixed                                                          developed lefthand hemyplegia. The patient has died. What
changes of a brain have caused development lefthand              E.      The type V
hemyplegia?
A.      Infarction of a brain                                    207. The 35yearold female was admitted into the hospital
B.      Hematoma.                                                with nephrolithiasis. Anaphylactic shock was appeared after
C.      Atherosclerosis of brain’s vessels.                      intravenous injection of contrast medium. Which cells take
D.      Edema of a brain.                                        part in development of this reaction?
E.      Meningitis.                                              A.        Tissue’s basophils
                                                                 B.        Fibroblasts
187. At histologic examination of tuberculoma resected           C.        Epitheliocytes
from right lung a necrosis in center is found out. Call a kind   D.        Myocytes
of the necrosis.                                                 E.        Neutrophils
A.           Caseous
B.           Fatty                                               208. Histologic investigation of thyroid gland has showed
C.           Liquefactive                                        destruction and atrophy of follicles, diffuse lymphoid
D.           Coagulative                                         infiltration with formation of lymphoid follicles in the
E.           Gangrene                                            stroma. Call the group of diseases with respect to this
                                                                 thyroiditis.
194. A woman, which was in a bed in forced position for a        A.         Autoimmune specific
long time, has died because of a tumoral dissemination           B.         Autoimmune nonspecific
(multiple metastases and carcinomatosis of a peritoneum) of      C.         Bacterial
mucinous cystadenocarcinoma. During the autopsy the              D.         Infectiousallergic
large areas of necrosis of the skin and underlying soft          E.         Viruses infectious
tissues of sacral part were found out. Call a kind of a
necrosis.                                                        209. Ultrastructural investigation of tissue’s biopsy has
A.          Bedsore                                              showed considerable increase of lyzosomes in the
B.          Infarct                                              cytoplasm of macrophages within the inflammative
C.          Sequester                                            infiltrate. What is connected with appearance of this
D.          Caseous necrosis                                     morphologic process?
E.          Waxylike (Zenker’s) necrosis                         A.         Activation of phagocytosis
                                                                 B.         Proliferation of cells
198. The examination of the child with measles showed the        C.         Torpid phagocytosis
nonclear border edematous fluctuated areas of redblack           D.         Activation of apoptosis
color in the soft tissues of the cheeks and perineum. What       E.         Torpid apoptosis
complication did develop in the child?
A.           Wet gangrene (noma)                                 210. In biopsy of stomach in patient with autoimmune
B.           Dry gangrene                                        gastritis it was found out: infiltration by lymphocytes and
C.           Gas gangrene                                        macrophages in mucous layer. Which type of
D.           Bedsore                                             hypersensitivity is connected with these morphologic
E.           Trophic ulcer                                       changes?
                                                                 A.        The type IV
202. The presence of the grains of chromatin in a focus of       B.        The type II
caseous necrosis is a manifestation of:                          C.        The type V
A.          Karyorhexis                                          D.        The type I
B.          Karyolysis                                           E.        The type III
C.          Karyopyknosis
D.          Mitotic activity of nuclei                           211. In biopsy of lymph node it was found out a lot of
E.          Apoptosis                                            lymphoid follicles with large centers of duplication,
                                                                 increasing of mitoses. Which process is characterized by
205. In 77yearold patient suffered with atherosclerosis the      these morphologic changes?
pain in the right foot has appeared. The foot is enlarged in     A.       Antigenic      stimulation   with     follicular
size, its skin has black color and is macerated; the                      hyperplasia
demarcation line is not clear. What pathological process         B.       Atrophy of lymphoid tissue
takes place in the patient?                                      C.       Lymphosarcoma
A. Wet gangrene                                                  D.       Hodgkin’s disease
B. Coagulative necrosis                                          E.       Metastases of cancer
C. Sequestrum
D. Dry gangrene                                                  212. It was found stopping of breathing and blood
E. Noma                                                          circulation in patient after traffic accident. Resuscitation
                                                                 was successful. Call the that condition of patient.
206. The 30yearold man has had for two months                    A.        Clinical death
lacrination, pruritic palpebras, rhinitis with mucus. All        B.        Social death
symptoms disappeared after treatment by desensebilizators.       C.        Biological death
What type of hypersensitivity occurred in patient?               D.        Physiologic death
A.        The type I                                             E.        Agony
B.        The type II
C.        The type III                                           213. Call the cells of the immune system that secrete
D.        The type IV                                            immunoglobulins:
A.      Plasma cells                                           Goodpasture’s syndrome. Which pathologic process is
B.      Macrophages                                            characteristic for this syndrome?
C.      Tlymphocytes                                           A.       Autoantibodies to basement membrane of the
D.      Dendritic cells                                                 lungs and kidneys
E.      Natural killer cells                                   B.       Autoantibodies to mitochondrias of the lungs and
                                                                        kidneys
214. Which cells are injured in patient with HIVinfection?     C.       Autoantibodies to DNA
A.      Tcells-helpers                                         D.       Cytotoxic reaction against epithelium of renal
B.      Blymphocytes                                                    tubules
C.      Natural killer cells                                   E.       Appearance of immune complexes in glomeruli of
D.      Macrophages                                                     kidneys and lungs
E.      Eosinophils
                                                               225. The patient was remove the firmly – elastic, fixed,
215. The 35yearold man has received prolonged                  painful tumor, it is localized in femor. At pathomorphologic
immunosupressive therapy after transplantation of kidney.      research it was found out: yellow tumor without the
He has died because of generalized cytomegaly. Call the        capsule, greyish with the locuses of necrosis and
kind of immunodeficiency in patient.                           hemorrhages; histologically is marked the expressed
A.       Medicamentous                                         polymorphism of cells. What is your diagnosis?
B.       Xradiating                                            A. Liposarcoma
C.       Bacterial                                             B. Rhabdomyosarcoma
D.       Primary                                               C. Angiosarcoma
E.       Ecological                                            D. Lipoma
                                                               E. Malignant synovioma
216. The 30yearold patient with transplanted kidney has
received prolonged immunosupressive therapy and he has         226. Under microscopic examination of the increased
died because of intoxication. Microscopic examination          cervical lymph node it was found out: the absence of its
showed giant cells with large nuclei encircled by ringslike    structure and the lymphoid follicles, infiltration by spherical
brightening, which looked as “owleye”, located in the          cells with narrow cytoplasm. It is known from the clinical
kidneys, liver, pancreas, lungs. Call this disease.            dates, that other groups of lymph nodules, and lien and liver
A.       Cytomegalovirus infection                             are increased also. What disease it is necessary to think
B.       Tuberculosis                                          about?
C.       Syphilis                                              A. Lymphoid leukosis
D.       Leprosy                                               B. Lymphogranulomatosis
E.       Bubonic plaque                                        C. Lymphosarcoma
                                                               D. Myeloid leucosis
217. In biopsy of the lymph node it was found out              E.    Myeloma
granuloma with epithelioid and giant cells. Call the origin
of epithelioid cell’s development.                             227. During autopsy of the body of the 30yearsold man has
A.       Monocytes                                             been determined the increased lien (weight 900,0),
B.       Pericytes                                             increased liver (weight 4000,0), increased lymph nodes.
C.       Neutrophils                                           Bone marrow of a diaphysis of a femur was red color.
D.       Lymphocytes                                           Microscopically: infiltration of the portal tract of liver by
E.       Epithelium cells                                      undifferentiated hemopoietic cells with the spherical shape
                                                               and narrow cytoplasm. What disease it is possible to think
222. In biopsy of transplantanted kidney it was found out:     about?
diffuse infiltration of stroma by lymphocytes, plasma cells,   A. Chronic lymphoid leucosis
lymphoblasts, plasmablasts, necrotic arteriitis. Which         B. Chronic myeloid leucosis
pathologic process was appeared in organ?                      C. Generalized form of lymphogranulomatosis
A.       Immune mutilation                                     D. Acute myeloblastic leucosis
B.       Acute glomerulonephritis                              E. Acute lymphoblastic leucosis
C.       Ischemic infarction
D.       Tuberculosis                                          228. At research of a biopsy of the increased cervical lymph
E.       Acute pyelonephritis                                  node it were found: the absence of structure of lymph
                                                               nodes, infiltration by proliferated lymphocytes, single
223. Pest’s pneumonia and croupous pneumonia in stage of       Shternberg’s cells. What do these changes testify about?
red hepatization may be differentiated due to presence of      A. Lymphogranulomatosis with predominance of
following elements in the lungs:                                   lymphoid tissue,
A.       Fibrin                                                B. Lymphogranulomatosis with suppression of lymphoid
B.       Carnification                                             tissue
C.       Macrophages                                           C. Mixed cellular variant of lymphogranulomatosis
D.       Red blood cells                                       D. Nodularsclerotic variant of lymphogranulomatosis
E.       Edema                                                 E. Lymphosarcoma

224. In the 30yearold woman it was found: cough, sputum        229. In biopsy of the increased cervical lymph node of a 14
with blood, fever, increased blood pressure, decreased urine   yearold girl the pathologist has found out: the absence of
output, edema of low extremities. All symptoms have            structure of lymph nodes, the absence of lymphoid follicles,
developed for 6 weeks. Diagnostic renal biopsy showed          foci of necrosis and sclerosis, cellular polymorphism.
There are lymphocytes, eosinocytes, atypical multinuclear       distributed irregular. Described morphological features are
cells of the large sizes and uninuclear cells of large sizes    characteristic for…:
also. Call this disease.                                        a. Fibroma
A. Lymphogranulomatosis                                         B. Rabdomyoma
B. Acute lymphoid leucosis                                      C. Papilloma
C. Chronic lymphoid leucosis                                    D. Polyps
D. Berkit’s lymphoma                                            E. Lipoma
E. Diffuse nonhodgkin’s lymphoma
                                                                235. In an autopsy of the 60yearold woman pathologist
230. A 14 yearold boy was found out: the increase of lower      has found out: tumor of uterine with growth surrounded
third of femur, local hyperemia, strengthening venous           tissue, enlarged regional lymph nodule, hemoperitoneum.
structure in a zone of pathological process. Ro: injury of      Histologically: cellular polymorphism and tissue atypism
supperficial layer of a femoral bone, joint are saved. In       in the endomtrium, necrosis and hemorrhages,
biopsy was found out the accumulation of atypical               metastasises of adenocarcinoma of lymph nodules. What
osteoblasts with multiple mitosises and foci of the             are secondary appearances of tumors?
anomalously formed osteal girders with an invasion in           1. Necrosis and hemorrhage
surraunded tissues. Call this disease.                          B. Lymphogenous metastasises
1. Osteogenic a sarcoma                                         C. Hematogenous metastasises
B. Chondrosarcoma                                               D. Cellular polymorphism
C. Fibrous dysplasia                                            E. Tissue atypism
D. Osteoid osteoma
E. Osteoblastoclastoma                                          236. In young 18yearold woman the sharp pain during
                                                                swallowing, enlargement of lymph nodes of the neck and
231. A 40 yearold man has been suffered by a chronic            increasing of temperature to 39 degrees of Centigrade have
bronchitis has died because of cachexy. During the autopsy      appeared. The whitishyellowish pellicles have appeared on
it was found: in a lumen of bronchuses the endophytic           the surface of tonsil’s mucosa. After separation of these
growth of light grayish soft tissue. Under the microscopic      pellicles the ulceration of tissue was formed. The state of
examination in a wall of a bronchus among growths of an         health of the patient has become progressively worse. She
atypical squamous epithelium were found the keratinous          has died in 8 days of the disease with signs of progressive
pearls. What is your diagnosis?                                 heart insufficiency. What histological changes in
1. Squamous carcinoma with a keratinization                     cardiomyocytes can be found?
B. Squamous carcinoma without a keratinization                  A. Fatty degeneration;
C. Adenocarcinoma                                               B. Mucousal degeneration;
D. Undifferentiated carcinoma                                   C. Hydropic degeneration;
E. Apudoma                                                      D. Balloon degeneration;
                                                                E. Hyalinedroplets degeneration.
232. At intraoperation biopsy of a female brest it was
found out: the concentric growth intralobular connective        237. A 55yearold woman has had fever and dyspnea for
tissue around of smallsized duct covered by a singlelayer       over a month along with a 2 kg weight loss. On physical
epithelium Your presumable diagnosis.                           examination her temperature is 37.8 C. A chest radiograph
A. Fibroadenoma                                                 shows a reticulonodular pattern along with prominent hilar
B. Intraductal papilloma                                        lymphadenopathy. A transbronchial lung biopsy is
C. Lobular carcinoma in sity                                    performed, and microscopic examination shows no viral
D. Noninvasive intraductul carcinoma                            inclusions, no fungi, no acid fast bacilli, and no atypical
E. Mastitis                                                     cells. Which of the following diseases is she most likely to
                                                                have?
233. At gastroscopical research of patient in area of           A. Silicosis
lesser curvature of the stomach it was found out the            B. Sarcoidosis
exophytic formation (1,5 cm in diameter). In biopsy             C. Asbestosis
from form it is marked : the growth of atypical                 D. Tuberculosis
glandular structures of different shape and size                E. Usual interstitial pneumonitis
(stretched, round, with irregular outlines, small or
large) into mucosa and submucosa; the epithelial                238. A 60yearold man has had a cough without production
cells are atypical too: their nuclei are polymorphous           of much sputum for the past week. On physical examination
with various maintenance of chromatin and mitoses.              he is afebrile. There are decreased breath sounds at the right
What is your diagnose?                                          lung base. A chest radiograph reveals an area of
A. Adenocarcinoma of stomach                                    consolidation in the right lower lobe. He is given antibiotic
B. Squamous carcinoma of stomach                                therapy, but a month later the radiographic picture has not
C. Polyp of stomach                                             changed, and his cough continues. A bronchoalveolar
D. Fibrous carcinoma (Skyrrus)                                  lavage is performed and yields atypical cells along with
E. Chronic ulcer                                                scattered alveolar macrophages. Which of the following is
                                                                the most likely diagnosis?
234. In the skin it was found out well –encapsulated firm       A. Mycoplasma pneumonia
node. On the cut it is showed whitish laminated tissue.         B. Bronchioloalveolar carcinoma
Histologically: the welldifferentiated connective tissue, the   C. Sarcoidosis
bands of fibers lay in different directions, vessels are        D. Pulmonary infarction
                                                                E. Silicosis
test is negative. By which of the following immunologic
239. Following a vehicular accident with blood loss leading     mechanisms is her disease most likely produced?
to prolonged, severe hypotension, a 30yearold man is            A. Local immune complex formation
intubated and placed on a ventilator. He has progressively      B. Interleukin release from macrophages
decreasing oxygen saturations despite increasing PEEP and       C. Binding of antireceptor antibody
FI02 of 100%. He remains afebrile. He dies 3 days later. At     D. Mast cell degranulation
autopsy, the lungs show diffuse hyaline membranes in            E. Crossreactivity with tissue antigens
alveoli, thickened alveolar walls, and many alveolar
macrophages but few neutrophils. Which of the following         252. A bee sting is suffered by a 28yearold man, and his
pulmonary diseases most likely complicated his course?          wife searches frantically for the medical kit with the
A. Bronchopneumonia                                             injectible epinephrine. Which of the following immunologic
B. Chronic bronchitis                                           mechanisms are they trying to prevent?
C. Bronchiectasis                                               A. Local immune complex formation
D. Viral pneumonia                                              B. Interleukin release from macrophages
E. Diffuse alveolar damage                                      D. Systemic anaphylaxis
                                                                C. Binding of antireceptor antibody
240. A 66yearold man has had increasing malaise for the         E. Complement activation
past year. On physical examination auscultation of the chest
reveals a friction rub. Laboratory studies show a serum urea    253. A 9yearold boy has a sore throat. A throat culture
nitrogen of 100 mg/dl and creatinine of 9.8 mg/dl. Which of     grows group A hemolytic streptococcus. 17 days later
the following forms of pericarditis is he most likely to        develops darkcoloured urine.       A renal biopsy was
have?                                                           performed. On immunofluoresence staining the biopsy
A. Fibrinous                                                    shows granular deposition of IgG and complement around
B. Hemorrhagic                                                  glomerular capillary loops. Which of the following immune
C. Purulent                                                     hypersensitivity mechanisms is most likely responsible for
D. Serous                                                       this pattern of findings?
E. Constrictive                                                 A. Type I
                                                                B. Type II
244. A 45yearold man was rushed to the hospital following       C. Type III
the sudden onset of an episode of crushing substernal chest     D. Type IV
pain. He receives advanced life support measures. His           E. Type V
course was marked by intractable cardiogenic shock and he
died 4 days later. At autopsy, a large transmural               254. A 48yearold man has had a chronic cough with fever
anterolateral area of coagulative necrosis was found in the     for 2 months. On physical examination his temperature is
anterolateral wall of the left ventricle. Which of the          37.9 C. A chest radiograph reveals a diffuse bilateral
following microscopic findings is most likely to be present?    reticulonodular pattern. A transbronchial biopsy is
A. Fibroblasts and collagen                                     performed. On microscopic examination of the biopsy there
B. Granulation tissue                                           are focal areas of inflammation containing epithelioid
C. Necrotic muscle and neutrophils                              macrophages, Langhans giant cells, and lymphocytes.
D. Granulomatous inflammation                                   These findings are most typical for which of the following
E. Diffuse chronic inflammation                                 immunologic responses?
                                                                A. Type I hypersensitivity
250. A 41yearold man has a history of drinking 1 to 2 liters    B. Type II hypersensitivity
of whisky per day for the past 20 years. He has had             C. Graft versus host disease
numerous episodes of nausea and vomiting in the past 5          D. Polyclonal Bcell activation
years. He experiences a bout of prolonged vomiting,             E. Type IV hypersensitivity
followed by massive hematemesis. On physical
examination in the emergency room, he has vital signs with      255. Twelve hours after going on a hike through dense
T 36.8 C, P 110, RR 22, and BP 80/40 mm Hg. His heart           foliage, a 40yearold man notices a slightly raised and tender
has a regular rate and rhythm with no murmurs and his           irregular reddish rash on one forearm that was not covered
lungs are clear to auscultation. There is no abdominal          by clothing. This rash gradually increases in intensity for 2
tenderness or distension and bowel sounds are present. His      days and then fades after two weeks. Which of the
stool is negative for occult blood. Which of the following is   following forms of hypersensitivity is most likely
the most likely diagnosis?                                      demonstrated in this patient?
A. Esophageal stricture                                         A Type I hypersensitivity
B. Esophageal laceration (MalloryWeiss syndrome)                B Type II hypersensitivity
C. Esophageal pulsion diverticulum                              C Type III hypersensitivity
D. Barrett esophagus (metaplasia with gastric mucosa)           D Type IV hypersensitivity
E. Esophageal squamous cell carcinoma                           E. Type V hypersensitivity

251. A 45yearold woman has experienced progressive,             256. The examination of ovarium tumor was performed. On
increasing muscular weakness, particularly toward the end       gross inspection of the mass, that was surgically excised,
of the day, over the past 2 months. She does not have           the surface of the mass is smooth, is not adherent to
arthralgias or myalgias. On physical examination her motor      surrounding pelvic structure, and is cystic and filled with
strength goes from 5/5 to 4/5 with repetitive movement of       hair on sectioning. On microscopic examination there is
extremities. A chest CT scan reveals an anterior mediastinal    squamous epithelium, tall columnar glandular epithelium,
mass. Laboratory studies show that her antinuclear antibody
cartilage, and fibrous connective tissue. Which of the         D Increased nuclear/cytoplasmic ratio
following neoplasms is she most likely to have?                E Expansive type of growth
A Teratoma
B Choristoma                                                   262. A clinical study of patient with pharyngeal infections
C Hamartoma                                                    is performed. Patient experienced fever and chills. On
D Myxoma                                                       physical examination, the most common finding is a
E Mesothelioma                                                 pharyngeal purulent exudate. Which of the following types
                                                               of inflammation did these patient most likely have?
257. A 44yearold woman who has had multiple sexual             A. Acute inflammation
partners for the past 30 years has an abnormal Pap smear       B. Granulomatous inflammation
with cytologic changes suggesting human papillomavirus         C. Abscess formation
infection. Without treatment, she is most likely to develop    D. Resolution of inflammation
which of the following lesions?                                E. Chronic inflammation
A Squamous cell carcinoma
B NonHodgkin's lymphoma                                        263. An empyema is a kind of inflammation as:
C Kaposi's sarcoma                                             A Purulent inflammation
D Adenocarcinoma                                               B Serous inflammation
E Leiomyoma                                                    C Fibrinous inflammation
                                                               D Chronic inflammation
258. A 41yearold woman has an exophytic 3 cm mass              E Granulomatous inflammation
involving the ectocervix. Pap smear testing is performed;
she has never had a previous Pap smear. Cytologic changes      264. A 45yearold man has had a fever and dry cough and
that are seen in Pap smears reveals squamous cell              his temperature is 38.5 C during last 3 days. A chest
carcinoma. Her serum glucose is 157 mg/dL. She has been        radiograph shows a right pleural effusion. A right
a commercial sex worker in the past. Which of the              thoracentesis is performed. The fluid obtained has a cloudy
following is the most likely risk factor for her cervical      appearance with a cell count showing 5500 leukocytes per
carcinoma?                                                     microliter, 98% of which are neutrophils. Which of the
A Human papillomavirus infection                               following terms best describes his pleural process?
B Diabetes mellitus, type II                                   A Purulent inflammation
C Heavy cigarette smoking                                      B Serous inflammation
D Pelvic inflammatory disease                                  C Fibrinous inflammation
E Previous cancer chemotherapy                                 D Chronic inflammation
                                                               E Granulomatous inflammation
259. A 27yearold woman in excellent health has a 2 cm
firm, rounded mass is palpable beneath the skin of the left     278. What type of exudates appears in pericardium cavity at
forearm. She has no difficulty using the arm and there is no    a rheumatic pericarditis?
associated pain with the mass, either in movement or on        A. Putrefactive
palpation. The overlying skin appears normal. The mass         B. Serousfibrinous
does not change in size over the next year. Which of the       C. Hemorrhagic
following neoplasms is she most likely to have?                D. Purulent
A Lipoma                                                       E. Catarrhal
B Metastatic carcinoma
C Melanoma                                                     284. A 90yearold woman has pneumonia. Two weeks later
D Rhabdomyosarcoma                                             a chest radiograph reveals a 3 cm rounded density in the
E Leiomyosarcoma                                               right lower lobe whose liquefied contents form a central
                                                               airfluid level and has a capsule. Which of the following is
260. A 35yearold woman had a firm nodule palpable on the       the best description for this outcome of her pneumonia?
dome of the uterus six years ago recorded on routine           A Abscess formation
physical examination. The nodule has slowly increased in       B Hypertrophic scar
size and is now appears to be about twice the size it was      C Regeneration
when first discovered. She remains asymptomatic. Which of      D Bronchogenic carcinoma
the following neoplasms is she most likely to have?            E Progression to chronic inflammation
A Leiomyoma
B Adenocarcinoma                                               285. A clinical study is peformed of patients with
C Leiomyosarcoma                                               pharyngeal infections. The most typical clinical course
D Hematoma                                                     averages 3 days from the time of onset until the patient sees
E Metastasis                                                   the physician. Most of these patients experienced fever and
                                                               chills. On physical examination, the most common finding
261. A study is performed to analyze characteristics of        is a pharyngeal purulent exudate. Which of the following
malignant neoplasms in biopsy specimens. The biopsies          types of inflammation did these patients most likely have?
were performed on patients who had palpable mass lesions       A Acute inflammation
on digital rectal examination. Which of the following          B Granulomatous inflammation
microscopic findings are most likely to indicate that the      C Abscess formation
neoplasm is malignant?                                         D Resolution of inflammation
A Invasion                                                     E Chronic inflammation
B Tissue pleomorphism
C Tissue atypia                                                286. At 38 yearold woman it is observed upperclavicle
multiplied lymphatic nodules. During biopsy research in the    gastritis has experienced weight loss and nausea for the past
lymphatic nodules a metastasis of fingerringcellular cancer    8 months. He does not have vomiting or diarrhea. In the
is exposed. What organ cancer most probably did develop        antrum of stomach endoscopy reveals an ulcerative mass
at the patient?                                                with elevated margins. Patient undergoes gastrectomy, and
A Cancer of stomach                                            the gross appearance shows ulcerated lesion with necrosis
B Cancer of thyroid                                            in the center that has extended below the submucosa, into
C Cancer of lungs                                              the muscularis, and has spread more widely extends into
D Cancer of gullet                                             the muscularis propria and beyond. Identify the type of
E Cancer of uterus                                             lesions:
                                                               A. Early gastric carcinoma
289. A 70yearold male with a long history of chronic           B. Advanced gastric carcinoma
alcoholism has had increasing difficulty with swallowing       C. Duodenum carcinoma
for the past 2 months. Upper endoscopy reveals an              D. Acute gastric ulcer
ulcerative midesophageal, 3cm mass that partially occludes     E. Chronic gastric ulcer
the esophageal lumen. He undergoes esophagectomy, and
the gross appearance shows ulcerated lesion with heaped up     237. An upper gastrointestinal radiographic series reveals
margins and squamous cell atypia of the esophageal             advanced gastric carcinoma in a 53yearold female who has
mucosa. Identify the type of lesions:                          had nausea, vomiting, and midepigastric pain for several
A. Leiomyosarcoma                                              months. Ultrasound investigation reveals an solid tumorous
B. Squamous cell carcinoma                                     enlargements of both ovaries 6x5 cm mass. Identify the
C. Dense collagenous scar                                      type of lesions in ovaries:
D. Adenocarcinoma                                              A. NonHodgkin lymphoma
E. Thrombosed vascular channels                                B. Cystadenoma of ovaries
                                                               C. Virchow's node
230. An upper gastrointestinal radiographic series reveals     D. Krukenberg tumors
gastric outlet obstruction in a 53yearold female who has had   E. Cystadenocarcima of ovaries
nausea, vomiting, and midepigastric pain for several
months. Upper endoscopy reveals an ulcerated 3x4 cm            238. A 67yearold male with a history of brain hemorrhage
mass at the pylorus. The biopsy was performed. Which of        has had an episodes of hematemesis at the hospital during
the following neoplasms is most likely to be seen on biopsy    past 2 weeks after hemorrhage. In the mucous shell of the
of this mass?                                                  stomach body endoscopy reveals an ulcer 2.5cm in
A. NonHodgkin lymphoma                                         diameter, round form, with a brown base. Identify the type
B. Neuroendocrine carcinoma                                    of ulcer:
C. Squamous cell carcinoma                                     A. Curling's
D. Adenocarcinoma                                              B. Virchow's
E. Leiomyosarcoma                                              C. Chronic
                                                               D. Cushing's
231. A 67yearold male with a long history of Barrett           E. Krukenberg’s
esophagus has had increasing difficulty with swallowing for
the past 2 months. In the lower third part of the esophagus    240. During autopsy it was found out croupous pneumonia
upper endoscopy reveals an ulcerative esophageal mass that     in lower lobe of the left lung and about 500ml of
partially occludes the esophageal lumen. Patient undergoes     greenishyellow fluid in the pleural cavity. Microscopically
esophagectomy, and the gross appearance shows ulcerated        pleural liquid contained many neutrophils. Identify the
lesion with necrosis in the center. Identify the type of       complication of pneumonia:
lesions:                                                       A. Hemothorax
A. Leiomyosarcoma                                              B. Fibrinous pleuritis
B. Squamous cell carcinoma                                     C. Pneumothorax
C. Dense collagenous scar                                      D. Hydrothorax
D. Adenocarcinoma                                              E. Empyema of pleura
E. Thrombosed vascular channels
                                                               241. 56yearold patient has suffered from rightside
235. A barium swallow is performed in a 44yearold female       lowerlobar pneumonia with expectoration of mucus with
who has had nausea and vomiting for months. It is known        pus. During autopsy in 910 segments of the right lung the
that she had a stomach ulcer for many years.                   cavity with dense walls filled with purulent masses, was
Radiographically, there is marked dilation of the stomach      found. The whitish path comes from the cavity toward the
with "beaking" in the distal portion where marked luminal      radix of the lung. Microscopically it was established that
narrowing exists. A biopsy of the antrum shows ulcer           the cavity is separated from notchanged lung tissue with
formation with elevated margins and prominent submucosal       thin membrane, which consists of two layers:
fibrosis without inflammation, epithelium cell atypism. The    internalgranulation tissue, and external – connective tissue.
most likely cause for these findings is                        What diagnosis is more probable?
A. Chronic duodenum ulcer                                      A. Chronic abscess
B. Chronic gastric ulcer                                       B. Pulmonary gangrene
C. Acute gastric ulcer                                         C. Acute pulmonary abscess
D. Gastric carcinoma                                           D. Chronic pneumonia
E. Duodenum carcinoma                                          E. Bronchoectatic disease

236. A 72yearold male with a long history of chronic
file
file
file
file
file
file
file
file
file
file
file
file
file
file
file
file
file

More Related Content

What's hot

Krok 2 - 2014 (Pediatrics)
Krok 2 - 2014 (Pediatrics)Krok 2 - 2014 (Pediatrics)
Krok 2 - 2014 (Pediatrics)Eneutron
 
Krok 1 - 2015 (Path-Physiology)
Krok 1 - 2015 (Path-Physiology)Krok 1 - 2015 (Path-Physiology)
Krok 1 - 2015 (Path-Physiology)Eneutron
 
Krok 2 - 2015 Question Paper (General Medicine)
Krok 2 - 2015 Question Paper (General Medicine)Krok 2 - 2015 Question Paper (General Medicine)
Krok 2 - 2015 Question Paper (General Medicine)Eneutron
 
Krok 2 Medicine - 2017 Question Paper
Krok 2 Medicine - 2017 Question PaperKrok 2 Medicine - 2017 Question Paper
Krok 2 Medicine - 2017 Question PaperEneutron
 
MCQs Hemodynamic Disorders, Thromboembolic Disease & Shock by DR. ROOPAM JAIN
MCQs Hemodynamic Disorders, Thromboembolic Disease & Shock by DR. ROOPAM JAINMCQs Hemodynamic Disorders, Thromboembolic Disease & Shock by DR. ROOPAM JAIN
MCQs Hemodynamic Disorders, Thromboembolic Disease & Shock by DR. ROOPAM JAINDr. Roopam Jain
 
MCQs respiratory system
MCQs respiratory systemMCQs respiratory system
MCQs respiratory systemDOCTOR WHO
 
Krok 1 - 2014 (Path-Anatomy)
Krok 1 - 2014 (Path-Anatomy)Krok 1 - 2014 (Path-Anatomy)
Krok 1 - 2014 (Path-Anatomy)Eneutron
 
Krok 2 - 2013 (Surgery)
Krok 2 - 2013 (Surgery)Krok 2 - 2013 (Surgery)
Krok 2 - 2013 (Surgery)Eneutron
 
Pathology mcq
Pathology mcqPathology mcq
Pathology mcqnawrasmel
 
Krok 2 - 2014 (Gynecology)
Krok 2 - 2014 (Gynecology)Krok 2 - 2014 (Gynecology)
Krok 2 - 2014 (Gynecology)Eneutron
 
Step 1 PMDC paper compiled by Dr. Asif ali
Step 1 PMDC paper compiled by Dr. Asif aliStep 1 PMDC paper compiled by Dr. Asif ali
Step 1 PMDC paper compiled by Dr. Asif aliDr. Shadab Kamal
 
Gastrointestinal mcq
Gastrointestinal mcqGastrointestinal mcq
Gastrointestinal mcqRashed Hassen
 
Krok 2 - 2006 Question Paper (General Medicine)
Krok 2 - 2006 Question Paper (General Medicine)Krok 2 - 2006 Question Paper (General Medicine)
Krok 2 - 2006 Question Paper (General Medicine)Eneutron
 
Krok 2 Medicine - 2018 Question Paper
Krok 2 Medicine - 2018 Question PaperKrok 2 Medicine - 2018 Question Paper
Krok 2 Medicine - 2018 Question PaperEneutron
 
Krok 2 - 2014 Question Paper (General Medicine)
Krok 2 - 2014 Question Paper (General Medicine)Krok 2 - 2014 Question Paper (General Medicine)
Krok 2 - 2014 Question Paper (General Medicine)Eneutron
 

What's hot (20)

Krok 2 - 2014 (Pediatrics)
Krok 2 - 2014 (Pediatrics)Krok 2 - 2014 (Pediatrics)
Krok 2 - 2014 (Pediatrics)
 
Krok 1 - 2015 (Path-Physiology)
Krok 1 - 2015 (Path-Physiology)Krok 1 - 2015 (Path-Physiology)
Krok 1 - 2015 (Path-Physiology)
 
Krok 2 - 2015 Question Paper (General Medicine)
Krok 2 - 2015 Question Paper (General Medicine)Krok 2 - 2015 Question Paper (General Medicine)
Krok 2 - 2015 Question Paper (General Medicine)
 
64971 hemodynamic mcq from ghadeer
64971 hemodynamic mcq from ghadeer64971 hemodynamic mcq from ghadeer
64971 hemodynamic mcq from ghadeer
 
Krok 2 Medicine - 2017 Question Paper
Krok 2 Medicine - 2017 Question PaperKrok 2 Medicine - 2017 Question Paper
Krok 2 Medicine - 2017 Question Paper
 
MCQs Hemodynamic Disorders, Thromboembolic Disease & Shock by DR. ROOPAM JAIN
MCQs Hemodynamic Disorders, Thromboembolic Disease & Shock by DR. ROOPAM JAINMCQs Hemodynamic Disorders, Thromboembolic Disease & Shock by DR. ROOPAM JAIN
MCQs Hemodynamic Disorders, Thromboembolic Disease & Shock by DR. ROOPAM JAIN
 
MCQs respiratory system
MCQs respiratory systemMCQs respiratory system
MCQs respiratory system
 
Krok 1 - 2014 (Path-Anatomy)
Krok 1 - 2014 (Path-Anatomy)Krok 1 - 2014 (Path-Anatomy)
Krok 1 - 2014 (Path-Anatomy)
 
Krok 2 - 2013 (Surgery)
Krok 2 - 2013 (Surgery)Krok 2 - 2013 (Surgery)
Krok 2 - 2013 (Surgery)
 
Pathology mcq
Pathology mcqPathology mcq
Pathology mcq
 
Mcq liver
Mcq liverMcq liver
Mcq liver
 
Pathology of CNS Tumors - Quiz
Pathology of CNS Tumors - QuizPathology of CNS Tumors - Quiz
Pathology of CNS Tumors - Quiz
 
Krok 2 - 2014 (Gynecology)
Krok 2 - 2014 (Gynecology)Krok 2 - 2014 (Gynecology)
Krok 2 - 2014 (Gynecology)
 
Step 1 PMDC paper compiled by Dr. Asif ali
Step 1 PMDC paper compiled by Dr. Asif aliStep 1 PMDC paper compiled by Dr. Asif ali
Step 1 PMDC paper compiled by Dr. Asif ali
 
Gastrointestinal mcq
Gastrointestinal mcqGastrointestinal mcq
Gastrointestinal mcq
 
Hemolytic anemia ppt presentation
Hemolytic anemia ppt presentationHemolytic anemia ppt presentation
Hemolytic anemia ppt presentation
 
Krok 2 - 2006 Question Paper (General Medicine)
Krok 2 - 2006 Question Paper (General Medicine)Krok 2 - 2006 Question Paper (General Medicine)
Krok 2 - 2006 Question Paper (General Medicine)
 
Krok 2 Medicine - 2018 Question Paper
Krok 2 Medicine - 2018 Question PaperKrok 2 Medicine - 2018 Question Paper
Krok 2 Medicine - 2018 Question Paper
 
MCQs KIDNEY PATHOLOGY
MCQs KIDNEY PATHOLOGYMCQs KIDNEY PATHOLOGY
MCQs KIDNEY PATHOLOGY
 
Krok 2 - 2014 Question Paper (General Medicine)
Krok 2 - 2014 Question Paper (General Medicine)Krok 2 - 2014 Question Paper (General Medicine)
Krok 2 - 2014 Question Paper (General Medicine)
 

Viewers also liked

Esophagus /certified fixed orthodontic courses by Indian dental academy
Esophagus   /certified fixed orthodontic courses by Indian dental academy Esophagus   /certified fixed orthodontic courses by Indian dental academy
Esophagus /certified fixed orthodontic courses by Indian dental academy Indian dental academy
 
Musica filme nosso_lar
Musica filme nosso_larMusica filme nosso_lar
Musica filme nosso_larrjcacupe
 
Integroitumista vai eriytymistä? Maahanmuuttajaperheiden nuorten kotoutuminen
Integroitumista vai eriytymistä? Maahanmuuttajaperheiden nuorten kotoutuminenIntegroitumista vai eriytymistä? Maahanmuuttajaperheiden nuorten kotoutuminen
Integroitumista vai eriytymistä? Maahanmuuttajaperheiden nuorten kotoutuminenKelan tutkimus / Research at Kela
 
LaserSpeed: Not All Non-Contact Length and Speed Encoders Are Created Equal
LaserSpeed: Not All Non-Contact Length and Speed Encoders Are Created EqualLaserSpeed: Not All Non-Contact Length and Speed Encoders Are Created Equal
LaserSpeed: Not All Non-Contact Length and Speed Encoders Are Created EqualLaserSpeed
 
Sari Kehusmaa: Koho-ryhmän eväät omaishoidon kehittämiseen
Sari Kehusmaa: Koho-ryhmän eväät omaishoidon kehittämiseenSari Kehusmaa: Koho-ryhmän eväät omaishoidon kehittämiseen
Sari Kehusmaa: Koho-ryhmän eväät omaishoidon kehittämiseenKelan tutkimus / Research at Kela
 
Google Instant Search
Google Instant SearchGoogle Instant Search
Google Instant SearchClickDirecto
 
Roys birthday
Roys birthdayRoys birthday
Roys birthdayLoannWest
 
8710 Adoption - CA
8710 Adoption - CA8710 Adoption - CA
8710 Adoption - CAATD13
 
2011 Q1 UX Research Launch at Company X
2011 Q1 UX Research Launch at Company X2011 Q1 UX Research Launch at Company X
2011 Q1 UX Research Launch at Company XBethany Pickard
 
Vacation planner
Vacation plannerVacation planner
Vacation plannerDuha Mouti
 
All about me
All about meAll about me
All about megreen13
 
Katri Aaltonen ja Jaana Martikainen: Ehdotukset lääkekorvausmenojen säästötoi...
Katri Aaltonen ja Jaana Martikainen: Ehdotukset lääkekorvausmenojen säästötoi...Katri Aaltonen ja Jaana Martikainen: Ehdotukset lääkekorvausmenojen säästötoi...
Katri Aaltonen ja Jaana Martikainen: Ehdotukset lääkekorvausmenojen säästötoi...Kelan tutkimus / Research at Kela
 
Commune info#35-web
Commune info#35-webCommune info#35-web
Commune info#35-webATD13
 

Viewers also liked (20)

Esophagus /certified fixed orthodontic courses by Indian dental academy
Esophagus   /certified fixed orthodontic courses by Indian dental academy Esophagus   /certified fixed orthodontic courses by Indian dental academy
Esophagus /certified fixed orthodontic courses by Indian dental academy
 
Musica filme nosso_lar
Musica filme nosso_larMusica filme nosso_lar
Musica filme nosso_lar
 
Integroitumista vai eriytymistä? Maahanmuuttajaperheiden nuorten kotoutuminen
Integroitumista vai eriytymistä? Maahanmuuttajaperheiden nuorten kotoutuminenIntegroitumista vai eriytymistä? Maahanmuuttajaperheiden nuorten kotoutuminen
Integroitumista vai eriytymistä? Maahanmuuttajaperheiden nuorten kotoutuminen
 
LaserSpeed: Not All Non-Contact Length and Speed Encoders Are Created Equal
LaserSpeed: Not All Non-Contact Length and Speed Encoders Are Created EqualLaserSpeed: Not All Non-Contact Length and Speed Encoders Are Created Equal
LaserSpeed: Not All Non-Contact Length and Speed Encoders Are Created Equal
 
Sari Kehusmaa: Koho-ryhmän eväät omaishoidon kehittämiseen
Sari Kehusmaa: Koho-ryhmän eväät omaishoidon kehittämiseenSari Kehusmaa: Koho-ryhmän eväät omaishoidon kehittämiseen
Sari Kehusmaa: Koho-ryhmän eväät omaishoidon kehittämiseen
 
Google Instant Search
Google Instant SearchGoogle Instant Search
Google Instant Search
 
Roys birthday
Roys birthdayRoys birthday
Roys birthday
 
8710 Adoption - CA
8710 Adoption - CA8710 Adoption - CA
8710 Adoption - CA
 
Thamus
ThamusThamus
Thamus
 
Wordpress test
Wordpress testWordpress test
Wordpress test
 
2011 Q1 UX Research Launch at Company X
2011 Q1 UX Research Launch at Company X2011 Q1 UX Research Launch at Company X
2011 Q1 UX Research Launch at Company X
 
Vacation planner
Vacation plannerVacation planner
Vacation planner
 
All about me
All about meAll about me
All about me
 
Katri Aaltonen ja Jaana Martikainen: Ehdotukset lääkekorvausmenojen säästötoi...
Katri Aaltonen ja Jaana Martikainen: Ehdotukset lääkekorvausmenojen säästötoi...Katri Aaltonen ja Jaana Martikainen: Ehdotukset lääkekorvausmenojen säästötoi...
Katri Aaltonen ja Jaana Martikainen: Ehdotukset lääkekorvausmenojen säästötoi...
 
Risk Management Midterm
Risk Management MidtermRisk Management Midterm
Risk Management Midterm
 
çOklu zeka kurami
çOklu zeka kuramiçOklu zeka kurami
çOklu zeka kurami
 
Commune info#35-web
Commune info#35-webCommune info#35-web
Commune info#35-web
 
Tesol112
Tesol112Tesol112
Tesol112
 
Hyvinvoinnin eurooppalainen ja globaali hallinta
Hyvinvoinnin eurooppalainen ja globaali hallintaHyvinvoinnin eurooppalainen ja globaali hallinta
Hyvinvoinnin eurooppalainen ja globaali hallinta
 
Investeringar i barns humankapital
Investeringar i barns humankapitalInvesteringar i barns humankapital
Investeringar i barns humankapital
 

Similar to file

Krok 1 - 2014 Path-Anatomy Base (General Medicine)
Krok 1 - 2014 Path-Anatomy Base (General Medicine)Krok 1 - 2014 Path-Anatomy Base (General Medicine)
Krok 1 - 2014 Path-Anatomy Base (General Medicine)E_neutron
 
Krok 1 2014 - path anatomy
Krok 1   2014 - path anatomyKrok 1   2014 - path anatomy
Krok 1 2014 - path anatomyEneutron
 
Krok 1 - 2015 (Histology)
Krok 1 - 2015 (Histology)Krok 1 - 2015 (Histology)
Krok 1 - 2015 (Histology)Eneutron
 
Krok 1 Medicine - 2016 General Medicine
Krok 1 Medicine - 2016 General MedicineKrok 1 Medicine - 2016 General Medicine
Krok 1 Medicine - 2016 General MedicineEneutron
 
Krok 1 - 2014 Question Paper (General medicine)
Krok 1 - 2014 Question Paper (General medicine)Krok 1 - 2014 Question Paper (General medicine)
Krok 1 - 2014 Question Paper (General medicine)Eneutron
 
Krok 1 - 2007 Question Paper (Stomatology)
Krok 1 - 2007 Question Paper (Stomatology)Krok 1 - 2007 Question Paper (Stomatology)
Krok 1 - 2007 Question Paper (Stomatology)Eneutron
 
Krok 1 - 2015 Question Paper (General medicine)
Krok 1 - 2015 Question Paper (General medicine)Krok 1 - 2015 Question Paper (General medicine)
Krok 1 - 2015 Question Paper (General medicine)Eneutron
 
Krok 1 - 2007 Question Paper (General medicine)
Krok 1 - 2007 Question Paper (General medicine)Krok 1 - 2007 Question Paper (General medicine)
Krok 1 - 2007 Question Paper (General medicine)Eneutron
 
Krok 1 - 2011 Question Paper (General medicine)
Krok 1 - 2011 Question Paper (General medicine)Krok 1 - 2011 Question Paper (General medicine)
Krok 1 - 2011 Question Paper (General medicine)Eneutron
 
Krok 1 - 2014 Histology Base (General Medicine)
Krok 1 - 2014 Histology Base (General Medicine)Krok 1 - 2014 Histology Base (General Medicine)
Krok 1 - 2014 Histology Base (General Medicine)E_neutron
 
Krok 1 2014 - histiology
Krok 1   2014 - histiologyKrok 1   2014 - histiology
Krok 1 2014 - histiologyEneutron
 
Krok 1 - 2012 Question Paper (Stomatology)
Krok 1 - 2012 Question Paper (Stomatology)Krok 1 - 2012 Question Paper (Stomatology)
Krok 1 - 2012 Question Paper (Stomatology)Eneutron
 
Krok 1 - 2014 (Histology)
Krok 1 - 2014 (Histology)Krok 1 - 2014 (Histology)
Krok 1 - 2014 (Histology)Eneutron
 
Krok1 stomatology - 2016
Krok1   stomatology - 2016Krok1   stomatology - 2016
Krok1 stomatology - 2016Eneutron
 
Krok 1 - 2014 (Path-Physiology)
Krok 1 - 2014 (Path-Physiology)Krok 1 - 2014 (Path-Physiology)
Krok 1 - 2014 (Path-Physiology)Eneutron
 
ETAS_MCQ_05 dermatopathology
ETAS_MCQ_05 dermatopathologyETAS_MCQ_05 dermatopathology
ETAS_MCQ_05 dermatopathologyDerma202
 

Similar to file (20)

2005
20052005
2005
 
2006
20062006
2006
 
2007
20072007
2007
 
Krok 1 - 2014 Path-Anatomy Base (General Medicine)
Krok 1 - 2014 Path-Anatomy Base (General Medicine)Krok 1 - 2014 Path-Anatomy Base (General Medicine)
Krok 1 - 2014 Path-Anatomy Base (General Medicine)
 
Krok 1 2014 - path anatomy
Krok 1   2014 - path anatomyKrok 1   2014 - path anatomy
Krok 1 2014 - path anatomy
 
Krok 1 - 2015 (Histology)
Krok 1 - 2015 (Histology)Krok 1 - 2015 (Histology)
Krok 1 - 2015 (Histology)
 
Krok 1 Medicine - 2016 General Medicine
Krok 1 Medicine - 2016 General MedicineKrok 1 Medicine - 2016 General Medicine
Krok 1 Medicine - 2016 General Medicine
 
Krok 1 - 2014 Question Paper (General medicine)
Krok 1 - 2014 Question Paper (General medicine)Krok 1 - 2014 Question Paper (General medicine)
Krok 1 - 2014 Question Paper (General medicine)
 
2008
20082008
2008
 
Krok 1 - 2007 Question Paper (Stomatology)
Krok 1 - 2007 Question Paper (Stomatology)Krok 1 - 2007 Question Paper (Stomatology)
Krok 1 - 2007 Question Paper (Stomatology)
 
Krok 1 - 2015 Question Paper (General medicine)
Krok 1 - 2015 Question Paper (General medicine)Krok 1 - 2015 Question Paper (General medicine)
Krok 1 - 2015 Question Paper (General medicine)
 
Krok 1 - 2007 Question Paper (General medicine)
Krok 1 - 2007 Question Paper (General medicine)Krok 1 - 2007 Question Paper (General medicine)
Krok 1 - 2007 Question Paper (General medicine)
 
Krok 1 - 2011 Question Paper (General medicine)
Krok 1 - 2011 Question Paper (General medicine)Krok 1 - 2011 Question Paper (General medicine)
Krok 1 - 2011 Question Paper (General medicine)
 
Krok 1 - 2014 Histology Base (General Medicine)
Krok 1 - 2014 Histology Base (General Medicine)Krok 1 - 2014 Histology Base (General Medicine)
Krok 1 - 2014 Histology Base (General Medicine)
 
Krok 1 2014 - histiology
Krok 1   2014 - histiologyKrok 1   2014 - histiology
Krok 1 2014 - histiology
 
Krok 1 - 2012 Question Paper (Stomatology)
Krok 1 - 2012 Question Paper (Stomatology)Krok 1 - 2012 Question Paper (Stomatology)
Krok 1 - 2012 Question Paper (Stomatology)
 
Krok 1 - 2014 (Histology)
Krok 1 - 2014 (Histology)Krok 1 - 2014 (Histology)
Krok 1 - 2014 (Histology)
 
Krok1 stomatology - 2016
Krok1   stomatology - 2016Krok1   stomatology - 2016
Krok1 stomatology - 2016
 
Krok 1 - 2014 (Path-Physiology)
Krok 1 - 2014 (Path-Physiology)Krok 1 - 2014 (Path-Physiology)
Krok 1 - 2014 (Path-Physiology)
 
ETAS_MCQ_05 dermatopathology
ETAS_MCQ_05 dermatopathologyETAS_MCQ_05 dermatopathology
ETAS_MCQ_05 dermatopathology
 

Recently uploaded

On National Teacher Day, meet the 2024-25 Kenan Fellows
On National Teacher Day, meet the 2024-25 Kenan FellowsOn National Teacher Day, meet the 2024-25 Kenan Fellows
On National Teacher Day, meet the 2024-25 Kenan FellowsMebane Rash
 
Wellbeing inclusion and digital dystopias.pptx
Wellbeing inclusion and digital dystopias.pptxWellbeing inclusion and digital dystopias.pptx
Wellbeing inclusion and digital dystopias.pptxJisc
 
Accessible Digital Futures project (20/03/2024)
Accessible Digital Futures project (20/03/2024)Accessible Digital Futures project (20/03/2024)
Accessible Digital Futures project (20/03/2024)Jisc
 
NO1 Top Black Magic Specialist In Lahore Black magic In Pakistan Kala Ilam Ex...
NO1 Top Black Magic Specialist In Lahore Black magic In Pakistan Kala Ilam Ex...NO1 Top Black Magic Specialist In Lahore Black magic In Pakistan Kala Ilam Ex...
NO1 Top Black Magic Specialist In Lahore Black magic In Pakistan Kala Ilam Ex...Amil baba
 
General Principles of Intellectual Property: Concepts of Intellectual Proper...
General Principles of Intellectual Property: Concepts of Intellectual  Proper...General Principles of Intellectual Property: Concepts of Intellectual  Proper...
General Principles of Intellectual Property: Concepts of Intellectual Proper...Poonam Aher Patil
 
The basics of sentences session 3pptx.pptx
The basics of sentences session 3pptx.pptxThe basics of sentences session 3pptx.pptx
The basics of sentences session 3pptx.pptxheathfieldcps1
 
Interdisciplinary_Insights_Data_Collection_Methods.pptx
Interdisciplinary_Insights_Data_Collection_Methods.pptxInterdisciplinary_Insights_Data_Collection_Methods.pptx
Interdisciplinary_Insights_Data_Collection_Methods.pptxPooja Bhuva
 
ICT Role in 21st Century Education & its Challenges.pptx
ICT Role in 21st Century Education & its Challenges.pptxICT Role in 21st Century Education & its Challenges.pptx
ICT Role in 21st Century Education & its Challenges.pptxAreebaZafar22
 
How to Give a Domain for a Field in Odoo 17
How to Give a Domain for a Field in Odoo 17How to Give a Domain for a Field in Odoo 17
How to Give a Domain for a Field in Odoo 17Celine George
 
Exploring_the_Narrative_Style_of_Amitav_Ghoshs_Gun_Island.pptx
Exploring_the_Narrative_Style_of_Amitav_Ghoshs_Gun_Island.pptxExploring_the_Narrative_Style_of_Amitav_Ghoshs_Gun_Island.pptx
Exploring_the_Narrative_Style_of_Amitav_Ghoshs_Gun_Island.pptxPooja Bhuva
 
Understanding Accommodations and Modifications
Understanding  Accommodations and ModificationsUnderstanding  Accommodations and Modifications
Understanding Accommodations and ModificationsMJDuyan
 
Salient Features of India constitution especially power and functions
Salient Features of India constitution especially power and functionsSalient Features of India constitution especially power and functions
Salient Features of India constitution especially power and functionsKarakKing
 
Sensory_Experience_and_Emotional_Resonance_in_Gabriel_Okaras_The_Piano_and_Th...
Sensory_Experience_and_Emotional_Resonance_in_Gabriel_Okaras_The_Piano_and_Th...Sensory_Experience_and_Emotional_Resonance_in_Gabriel_Okaras_The_Piano_and_Th...
Sensory_Experience_and_Emotional_Resonance_in_Gabriel_Okaras_The_Piano_and_Th...Pooja Bhuva
 
Beyond_Borders_Understanding_Anime_and_Manga_Fandom_A_Comprehensive_Audience_...
Beyond_Borders_Understanding_Anime_and_Manga_Fandom_A_Comprehensive_Audience_...Beyond_Borders_Understanding_Anime_and_Manga_Fandom_A_Comprehensive_Audience_...
Beyond_Borders_Understanding_Anime_and_Manga_Fandom_A_Comprehensive_Audience_...Pooja Bhuva
 
Application orientated numerical on hev.ppt
Application orientated numerical on hev.pptApplication orientated numerical on hev.ppt
Application orientated numerical on hev.pptRamjanShidvankar
 
Fostering Friendships - Enhancing Social Bonds in the Classroom
Fostering Friendships - Enhancing Social Bonds  in the ClassroomFostering Friendships - Enhancing Social Bonds  in the Classroom
Fostering Friendships - Enhancing Social Bonds in the ClassroomPooky Knightsmith
 
FSB Advising Checklist - Orientation 2024
FSB Advising Checklist - Orientation 2024FSB Advising Checklist - Orientation 2024
FSB Advising Checklist - Orientation 2024Elizabeth Walsh
 
Graduate Outcomes Presentation Slides - English
Graduate Outcomes Presentation Slides - EnglishGraduate Outcomes Presentation Slides - English
Graduate Outcomes Presentation Slides - Englishneillewis46
 
Single or Multiple melodic lines structure
Single or Multiple melodic lines structureSingle or Multiple melodic lines structure
Single or Multiple melodic lines structuredhanjurrannsibayan2
 
This PowerPoint helps students to consider the concept of infinity.
This PowerPoint helps students to consider the concept of infinity.This PowerPoint helps students to consider the concept of infinity.
This PowerPoint helps students to consider the concept of infinity.christianmathematics
 

Recently uploaded (20)

On National Teacher Day, meet the 2024-25 Kenan Fellows
On National Teacher Day, meet the 2024-25 Kenan FellowsOn National Teacher Day, meet the 2024-25 Kenan Fellows
On National Teacher Day, meet the 2024-25 Kenan Fellows
 
Wellbeing inclusion and digital dystopias.pptx
Wellbeing inclusion and digital dystopias.pptxWellbeing inclusion and digital dystopias.pptx
Wellbeing inclusion and digital dystopias.pptx
 
Accessible Digital Futures project (20/03/2024)
Accessible Digital Futures project (20/03/2024)Accessible Digital Futures project (20/03/2024)
Accessible Digital Futures project (20/03/2024)
 
NO1 Top Black Magic Specialist In Lahore Black magic In Pakistan Kala Ilam Ex...
NO1 Top Black Magic Specialist In Lahore Black magic In Pakistan Kala Ilam Ex...NO1 Top Black Magic Specialist In Lahore Black magic In Pakistan Kala Ilam Ex...
NO1 Top Black Magic Specialist In Lahore Black magic In Pakistan Kala Ilam Ex...
 
General Principles of Intellectual Property: Concepts of Intellectual Proper...
General Principles of Intellectual Property: Concepts of Intellectual  Proper...General Principles of Intellectual Property: Concepts of Intellectual  Proper...
General Principles of Intellectual Property: Concepts of Intellectual Proper...
 
The basics of sentences session 3pptx.pptx
The basics of sentences session 3pptx.pptxThe basics of sentences session 3pptx.pptx
The basics of sentences session 3pptx.pptx
 
Interdisciplinary_Insights_Data_Collection_Methods.pptx
Interdisciplinary_Insights_Data_Collection_Methods.pptxInterdisciplinary_Insights_Data_Collection_Methods.pptx
Interdisciplinary_Insights_Data_Collection_Methods.pptx
 
ICT Role in 21st Century Education & its Challenges.pptx
ICT Role in 21st Century Education & its Challenges.pptxICT Role in 21st Century Education & its Challenges.pptx
ICT Role in 21st Century Education & its Challenges.pptx
 
How to Give a Domain for a Field in Odoo 17
How to Give a Domain for a Field in Odoo 17How to Give a Domain for a Field in Odoo 17
How to Give a Domain for a Field in Odoo 17
 
Exploring_the_Narrative_Style_of_Amitav_Ghoshs_Gun_Island.pptx
Exploring_the_Narrative_Style_of_Amitav_Ghoshs_Gun_Island.pptxExploring_the_Narrative_Style_of_Amitav_Ghoshs_Gun_Island.pptx
Exploring_the_Narrative_Style_of_Amitav_Ghoshs_Gun_Island.pptx
 
Understanding Accommodations and Modifications
Understanding  Accommodations and ModificationsUnderstanding  Accommodations and Modifications
Understanding Accommodations and Modifications
 
Salient Features of India constitution especially power and functions
Salient Features of India constitution especially power and functionsSalient Features of India constitution especially power and functions
Salient Features of India constitution especially power and functions
 
Sensory_Experience_and_Emotional_Resonance_in_Gabriel_Okaras_The_Piano_and_Th...
Sensory_Experience_and_Emotional_Resonance_in_Gabriel_Okaras_The_Piano_and_Th...Sensory_Experience_and_Emotional_Resonance_in_Gabriel_Okaras_The_Piano_and_Th...
Sensory_Experience_and_Emotional_Resonance_in_Gabriel_Okaras_The_Piano_and_Th...
 
Beyond_Borders_Understanding_Anime_and_Manga_Fandom_A_Comprehensive_Audience_...
Beyond_Borders_Understanding_Anime_and_Manga_Fandom_A_Comprehensive_Audience_...Beyond_Borders_Understanding_Anime_and_Manga_Fandom_A_Comprehensive_Audience_...
Beyond_Borders_Understanding_Anime_and_Manga_Fandom_A_Comprehensive_Audience_...
 
Application orientated numerical on hev.ppt
Application orientated numerical on hev.pptApplication orientated numerical on hev.ppt
Application orientated numerical on hev.ppt
 
Fostering Friendships - Enhancing Social Bonds in the Classroom
Fostering Friendships - Enhancing Social Bonds  in the ClassroomFostering Friendships - Enhancing Social Bonds  in the Classroom
Fostering Friendships - Enhancing Social Bonds in the Classroom
 
FSB Advising Checklist - Orientation 2024
FSB Advising Checklist - Orientation 2024FSB Advising Checklist - Orientation 2024
FSB Advising Checklist - Orientation 2024
 
Graduate Outcomes Presentation Slides - English
Graduate Outcomes Presentation Slides - EnglishGraduate Outcomes Presentation Slides - English
Graduate Outcomes Presentation Slides - English
 
Single or Multiple melodic lines structure
Single or Multiple melodic lines structureSingle or Multiple melodic lines structure
Single or Multiple melodic lines structure
 
This PowerPoint helps students to consider the concept of infinity.
This PowerPoint helps students to consider the concept of infinity.This PowerPoint helps students to consider the concept of infinity.
This PowerPoint helps students to consider the concept of infinity.
 

file

  • 1. PATHOMORPHOLOGY GENERAL 1. Histological examination of a 40 yo. man's thymus D. II revealed decreased share of parenchymatous gland E. V elements, increased share of adipose and loose connective tissue, its enrichment with thymus bodies. The organ's mass 11. A 22 y.o. woman has enlarged lymph nodes. was unchanged. What phenomenon is it? Histologically: a lymph node contains lymphocytes, A. Age involution histiocytes, reticular cells, small and big Hodgkin's cells, B. Accidental involution multinucleated Sternberg cells, isolated foci of caseous C. Hypotrophy necrosis. What disease are these changes typical for? D. Atrophy A. Lymphosarcoma E. Dystrophy B. Lymphogranulomatosis C. Chronic leukosis 4. A 9 m.o. child has delayed dentition, it is also out of D. Acute leukosis order. Upper jaw configuration is horizontal ("high"palate); E. Lung cancer metastasis microscopically irregular mineralization of tooth enamel, wrinkled enamel prisms, some of them are vacuolized. 12. A patient died under conditions of cardiovascular Predentin zone is extended; there are solitary denticles. insufficiency. Autopsy results: postinfarction cardiosclerosis, What disease is it? myocardium hypertrophy and dilatation of its cavities, A. Early rickets especially of its right ventricle. Liver is enlarged, its surface B. Hypervitaminosis D is smooth, incision revealed that it was plethoric, with C. Late rickets darkred specks against the background of brownish tissue. D. Gout Histologically: plethora of central parts of lobules; E. Osteomalacia peritheral parts around portal tracts contain hepatocytes in a state of adipose degeneration. How are these liver changes 6. A patient ill with diabetes mellitus felt acute pain in his called? right foot. Objectively: foot thumb is black, foot tissues are A. Pseudonutmeg liver edematous, there are foci of epidermis desquamation, B. Amyloidosis stinking discharges. What clinicopathological form of C. C. Nutmeg liver necrosis is it? D. Liver cirrhosis A. Infarction E. Liver steatosis B. Sequestrum C. Dry gangrene 13. In the surgical department of a hospital there was an D. Bedsore outbreak of hospital infecti on that showed itself in often E. Moist gangrene postoperative wound abscesses. Bacteriological examination of pus revealed aurococcus. What examination shall be 8. Autopsy of a man who died from influenza revealed that conducted to find out the source of this causative agent his heart was slightly enlarged, pastous, myocardium was among the department personnel? dull and had specks. Microscopical examination of A. Microscopical examination myocardium revealed signs of parenchymatous adipose and B. Phagotyping hydropic dystrophy; stroma was edematic with poor C. Estimation of antibiotic susceptibility macrophagal and lymphocytic infiltration, vessels were D. Biochemical identification plethoric; perivascular analysis revealed petechial E. Serological identification hemorrhages. What type of myocarditis was developed in this case? 17. Autopsy of a man who died from chronic cardiovascular A. Purulent collapse revealed "tiger heart". Sidewards of endocardium a B. Granulomatous yellowishwhite banding can be seen; myocardium is dull, C. Interstitial proliferative darkyellow. What process caused this pathology? D. Serous focal A. Fatty vascularstromal degeneration E. Serous diffuse B. Fatty parenchymatous degeneration C. Carbohydrate degeneration 9. Colonoscopy of a patient ill with dysentery revealed D. Hyaline degeneration that mucous membrane of his large intestine is hyperemic, E. Amyloidosis edematic, its surface was covered with greyandgreen coats. Name the morphological form of dysenteric collitis: 18. A 48 y.o. patient was admitted to the hospital with A. Fibrinous complaints about weakness, irritability, sleep disturbance. B. Necrotic Objectively: skin and scleras are yellow. In blood: C. Purulent conjugated bilirubin, cholalemia. Feces are acholic. Urine D. Ulcerous is of dark colour (bilirubin). What jaundice is it? E. Catarrhal A. Gilbert's syndrome B. Parenchymatous 10. A woman has been applying a new cosmetic C. Mechanic preparation for a week that resulted in eyelid inflammation D. Hemolytic accompanyed by hyperemia, infiltration and painfulness. E. CriglerNajjar syndrome What type of allergic reaction was developed? A. I 21. A 56 yo. patient has been suffering from thyreotoxicosis B. IV for a long time. What type of hypoxia can be developed? C. С III A. Tissue
  • 2. B. Mixed B. Pancreas C. Circulatory C. Thyroid D. Hemic D. Thymus E. Respiratory E. Liver 22. Microscopical examination of a removed appendix 32. Autopsy of a 58 y.o. man revealed that bicuspid valve revealed an edema, diffuse neutrophilic infiltration of was deformed, thickened and unclosed. Microscopically: appendix wall along with necrosis and defect of mucous foci of collagen fibrilla are eosinophilic, react positively membrane with affection of its muscle plate. What to fibrin. The most probably it is: appendicitis form was developed? A. Fibrinous inflammation A. Phlegmonous B. Hyalinosis B. Apostematous C. Amyloidosis C. Superficial D. Fibrinoid swelling D. Ulcerophlegmonous E. Mucoid swelling E. Gangrenous 34. A patient with clinical presentations of 23. A patient is ill with diabetes mellitus that is immunodeficiency went through immunological accompanied by hyperglycemia of over 7,2 millimole/l on examinations. They revealed significant loss of cells that an empty stomach. The level of what blood plasma protein form rosettes with erythrocytes of a ram. What conclusion allows to estimate the glycemia rate retrospectively (48 can be made according to the analysis data? weeks before examination)? A. Decrease of natural killer cell rate A. Glycated hemoglobin B. Insufficiency of effector cells of humoral immunity B. Ceruloplasmin C. Decrease of T-lymphocytes rate C. Creactive protein D. Decrease of complement system rate D. Fibrinogen E. Decrease of B-lymphocytes rate E. Albumin 35. Autopsy of a patient who suffered from croupous 24. A 45 y.o. patient consulted a doctor about pneumonia and died from pneumococcal sepsis revealed plaqueshaped formation on his neck. Histological 900 ml of turbid greenishyellow liquid in the right pleural examination of biopsy skin material revealed tumourous cavity. Pleural leaves are dull, plephoric. Name the cells of round and oval form with thin ring of basophilic clinicopathological form of inflammation in the pleural cytoplasma that resemble of cells of basal epidermal layer. cavity What tumour is it? A. Phlegmon A. Basalioma B. Chronic abscess B. Syringoadenoma C. Empyema C. Epidermal cancer D. Fibrinous inflammation D. Hydradenoma E. Acute abscess E. Trichoepithelioma 36. Mucous membrane of the right palatine tonsil has a 29. An experimental animal was first sensibilized painless ulcer with smooth lacquer fundus and regular whereupon an antigen dose was introduced subcutaneously. cartilaginous edges. Microscopically: inflammatory This injection resulted in the development of a fibrinous infiltration that consists of lymphocytes, plasmocytes, a inflammation with alteration of vessel walls, basal small number of neutrophils and epithelioid cells; substance and fibrous structures of connective tissue in endovasculitis and perivasculitis. What disease is it? form of mucoid and fibrinoid swelling and necrosis. What A. Pharyngeal diphtheria immunological reaction took place? B. Ulcerous necrotic Vincent's angina A. Delayed type hypersensitivity C. Actinomycosis B. Immediate hypersensitivity D. Syphilis C. Reaction of transplantation immunity E. Tuberculosis D. Normergic reaction E. Granulomatosis 37. A 27yearsold woman has been suffered by diabetes mellitus since she had been a child. During the last years 30. Examination of cell culture got from a patient with the arterial hypertension and proteinuria have occurred. She lysosomal pathology revealed accumulation of great has died with signs of uremia. What typical changes were quantity of lipids in the lysosomes. What of the following found out in autopsy? diseases is this disturbance typical for? A. “Cor villosum” A. Wilson disease B. “Tiger heart” B. Galactosemia C. “Armor heart” C. TaySachs disease D. Mesenchymal fatty dystrophy of the heart D. Phenylketonuria E. Purulent pericarditis E. Gout 38. A 43yearsold patient has a burn of right hand. The 31. A teenager was irradiated with high radiation dose that exfoliation of epidermis and formation of bubbles filled by resulted in serious damages of lymphoid system, lysis of semitransparent fluid is manifested in the palm and the back many lymphocytes. Restoration of normal hemogram is surface of the hand. What kind of inflammation occurs in possible due to the functioning of the following gland: that case? A. Adrenal A. A.Serous
  • 3. B. B.Purulent E. After anemia C. C.Fibrinous D. D.Putrefactive 45. A 23yearsold patient got a lesion of the liver because of E. E.Catarrhal trauma. In time a structure and functions of that organ was restored completely. What kind of regeneration did occur in 39. A patient was admitted in the hospital with diagnosis of that case? trichinellosis. What kind of inflammation develops within A. Restitution the muscles around the parasites? B. Pathologic regeneration A. Proliferative granulematous C. Physiologic regeneration B. Exudative fibrinous D. Metaplasia C. Exudative purulent E. Substitution D. Exudative serous E. Proliferative interstitial 46. A 4yearsold girl was operated because of an acute appendicitis. During the operation in the retroperitoneal 40. A 22yearsold patient was admitted in hospital with space it was found that the right kidney is less by 1/3 in complaints on heavy nasal breathing. During the comparison with the left one. The diameter of the right examination of nasal cavity it was found the thickened renal artery was 0,3 cm, the left one was 0,4 cm. What mucous membrane, a lot of mucus and nodular infiltrates pathologic process did occur in that case? without erosions. It was diagnosed as rhinoscleroma of A. Hypoplasia nose. It was took the biopsy. What typical morphological B. Pathologic atrophy changes may be found? C. Physiologic atrophy A. Granulomas with Mikulicz’s cells D. Aplasia B. Granulomas with Virchow’s cells E. Agenesia C. Granulomas with Langhan’s cells D. Granulomas with foreign body cells 47. A patient has suffered with mitral insufficiency that E. Interstitial inflammation complicated by chronic cardiac insufficiency with grossly marked edema of the lower extremities. Suddenly cyanosis 41. During the histological examination of the fragment of of the face is manifested with attempt standing up and he mucous membrane of urine bladder of the patient suffered has died. What the most probably complication developed with chronic cystitis it was found out: the transitional in patient? epithelium focally is replaced by stratified nonkeratinized A. Thromboembolism of the pulmonary artery epithelium. These changes can manifest development of: B. Venousarterial reaction A. Metaplasia C. Infarction of myocardium B. Dysplasia D. Chronic venous hyperemia C. Leukoplakia E. Hypertension stroke D. Hyperkeratosis E. Hyperplasia 48. A child fell down the tree and got a simple fracture of the one of hand’s bones. In a time healing occurred. Call the 42. A 1/3 of the liver with a multicellular echinococcus was kind of regeneration and cells, which taking part in resected during the operation. What typical changes of the restoration. liver tissue surrounding the parasites were found out by the A. Reparative regeneration, osteoclasts and osteoblasts pathologist during the histological examination of the B. Physiological regeneration, octeoclasts resected part of the organ? C. Pathological regeneration, osteoclasts A. Formation of nonspecific granulomas D. Intracellular regeneration, osteocytes B. Flabby consistence of the liver E. Cellular regeneration, osteoclasts and osteoclasts C. Formation of specific granulomas D. Exudative tissue reaction 49. A pilot has died because of depressurization of the cabin E. Hydropic dystrophy of the airplane. In autopsy it was found out a gross marked rigor mortis, emphysema of subcutaneous tissue of the body 43. During the autopsy it was found the enlarged firm liver and face. There were also edema and perivascular with rounded margins; the surface of the cut looks hemorrhages within the lungs. There was a foamy and fluid nutmeglike of yellowbrown colour with darkred spots. blood within the veins. The histological examination was What pathologic process do underlie of those changes? showed a lot of bubbles within the vessels of all inner A. Chronic venous hyperemia organs, a fat dystrophy of the liver and a quantity of small B. Arterial hyperemia gray softening areas in the brain. Call the more probable C. Arterial ischemia cause of those changes. D. Fat embolism A. Gas embolism E. Chronic bleeding B. Air embolism C. Tissue embolism 44. There is a redness of skin above the site of obturation, if D. Thromboembolism the thrombus obstructed the main artery of upper extremity. E. Fat embolism Call the kind of arterial hyperemia appearing as a result of disturbance of bloodstream. 50. A man has died in 8th day since beginning of the A. Collateral disease. It was diagnosed dysentery. During the autopsy it B. Angioneurotic was found out a thickened wall of the sigma and rectum, C. Inflammatory fibrinous membrane on the surface of mucous membrane. D. Physiological Histologically: there is a deep necrosis of mucous
  • 4. membrane with infiltration of necrotic masses with fibrin. C. Hyperplasia What kind of colitis does correspond to those changes? D. Neoplasm of kidney A. Diphtheritic E. Pyelonephritis B. Catarrhal C. Ulcerative 57. A woman suffered with dysfunctional metrorrhagia was D. Chronic made a diagnostic abortion. Histologically in the scrape E. Gangrenous there were a lot of small stamped glandulas covered by multirowed epithelium. The lumens of some glandulas were 51. A patient suffered with rheumatic fever with a mitral cystically extended. Call the variant of general pathologic valve disease of the heart for a long time. He has died as a process in the endometrium. result of cardiacpulmonary insufficiency. During the A. Glandularcystic hyperplasia of endometrium autopsy it was found out a brown induration of the lungs. B. Atrophy of endometrium Call kind of hemodynamic disorders causing those changes. C. Metaplasia of endometrium A. Chronic left ventricle failure D. Neoplasm of endometrium B. Chronic right ventricle failure E. Hypertrophic growth C. Acute left ventricle failure D. Acute right ventricle failure 58. A 13yearsold child was admitted into the hospital with E. Portal hypertension complaints of fever, weakness, and intense pain in the right thigh. From anamnesis it was defined that he has bitten by 52. A patient suffered by stenosis of mitral valve of 4th stage dog 3 days ago. During the visual examination it was found has died as a result of cardiacpulmonary insufficiency. out redness, swelling and acute painfulness along right Autopsy was showed a hemosiderosis of the lungs. Call the thigh. After excision it was made the histological kind of hemodynamic disorder in the lungs. examination, which was showed a considerable A. Local venous hyperemia accumulations of polymorphonucleus leukocytes between B. General venous hyperemia the mussel fibers partly undergone by purulent fusion. C. Arterial hyperemia Diagnose that condition. D. Inflammatory hyperemia A. Phlegmon E. Hemorrhage B. Abscess C. Gangrene 53. A patient has died as a result of cardiac insufficiency. In D. Empyema anamnesis he had got a pulmonectomy in account of cyst of E. Croupous inflammation the right lung. During the autopsy enlarged left one was found out. Call the pathologic process in the left lung. 59. A patient with severe fracture of both femoral bones has A. Replacement hypertrophy died in a 4 hours after trauma. What additional staining of B. Neurohumoral hypertrophy microscopic sections is it necessary to make for exacter C. Dyscirculatory atrophy diagnosis of the cause of death? D. Dysfunctional atrophy A. The lungs and brain by Sudan III E. Physiologic hypertrophy B. The lungs and kidney by Congored C. The lungs and liver by Perl’s reaction 54. A patient has undergone to amputation of lower D. The brain and kidney by Congored extremity. In a time a painful nodules appeared in a stump. E. The kidney and heart by toluidinblue Amputatious neuromas were found out during the microscopical examination. What kind of pathological 61. A 59yearsold woman was admitted into the hospital processes does those formations relate to? with signs of exicosis being in deep shock. After her death A. Regeneration it was made autopsy showed presence of vibrio cholerae, B. Dystrophy moderate mononuclear infiltrations in the mucosal lamina C. Inflammation propria of intestine. What kind of shock was the cause of D. Hyperemia death of the patient? E. Metaplasia A. Hypovolemia B. Cardiogenic 55. After deep burns of the skin a patient has got a keloid C. Anaphylactic scarring. What kind of pathologic processes does those D. Traumatic formations relate to? E. Toxic A. Incomplete regeneration (substitution) B. Complete regeneration (restitution) 62. A cattle worker has died with signs of severe C. Atrophy intoxication in a 2nd day after beginning of disease. Autopsy D. Hypertrophy was showed the enlarged flabby spleen, on the surface of E. Metaplasia the cut of darkcherry coloured; the scrape of the pulp is abundant. The leptomeninges are swelling, infiltrating by 56. A patient suffered with urolithiasis has died as a result blood of darkred colour (“a red cap”). It was defined a uremia. Autopsy is showed an enlargement of right kidney, microorganism – bacillus anthracis – and diagnosed its parenchyma has been thinned; the pelvis and calices anthrax. What kind of inflammation does occur in that have been extended, filled with fluid. In the orifice of ureter disease? there has been a calculus. Call the pathologic process in the A. Hemorrhagic right kidney. B. Phlegmonous A. Hydronephrosis C. Fibrinous B. Replacement hypertrophy D. Purulent
  • 5. E. Putrefactive D. Tuberculosis. E. Actinomycosis. 63. A 65yearsold patient suffered by thrombophlebitis of the deep veins of both legs has died suddenly. Autopsy was 69. The patient with severe intoxication has died. During showed free lying dry friable red masses with a dull the autopsy it was found out: the left tonsil was enlarged, crimped surface within the truncus pulmonalis and firm; in section it contained a lot of cavities filled by bifurcation of the lung artery. What process within the yellow–greenish masses. The tissues of the neck, and vessels did pathologist find? mediastinum were undergoing with suppurative fusion. A. Thromboembolism Microscopically: there were a lot of abscesses, in the center B. Thrombosis of which intensive basophilic formations were defined. C. Tissue embolism Those formations consist of short sticklike elements, D. Postmortem clot surrounded with radiating projections (“ray fungus”). Such E. Hemangioma changes characterized: A. Actinomycosis. 64. A patient was admitted into the hospital with complaints B. Amebiasis. to feel sick, pains in the epigastria after meal. During the C. Brucellosis. gastroscopy it was found out hyperemia, edema of mucous D. Tuberculosis. membrane of the stomach, an excessive amount of viscous E. Scarlet fever. grayish mucus. What kind of inflammation did develop in the stomach? 70. A 34yearsold woman has got pain, redness and swelling A. Catarrhal of finger on account of careless use of iron. The blister B. Fibrinous filled by transparent yellowish fluid appears in some C. Serous minutes. What pathologic process do such signs D. Hemorrhagic characterize? E. Purulent A. Exudative inflammation. B. Traumatic edema. 65. During the autopsy it was found out: the lungs are firm, C. Hypertrophy. have brown colour because of accumulation of endogenous D. Proliferative inflammation. pigment. It’s known that the patient has suffered with E. Hydropic dystrophy. venous congestion in the pulmonary circulation. What process did cause such changes? 71. The patient with chronic glomerulonephritis has died A. Hemosiderosis with accompanied signs of uremia. The cadaver had B. Calcinosis specific uremic smell. Macroscopically: there were a lot of C. Jaundice pilelike whitishgrayish puttings on the surface of D. Melanosis pericardium. The vessels filled with blood became visible E. Porphiria after removal of such formations. What process did develop in the pericardium? 66. During the autopsy it was found out the cavity 2,5x1,5 A. Fibrinous inflammation. sm within the right frontal part of the brain. It was filled B. Organization. with transparent fluid and its walls were smooth and of C. Proliferative inflammation. brownish colour. What process did develop in the brain? D. Hemorrhagic inflammation. A. Cyst as an outcome of hemorrhage. E. Arterial hyperemia. B. Grey softening of brain. C. Abscess of brain. 72. The patient complains of local pain in the back of the D. Birth defect of brain. head, increasing of the temperature in that area. E. Cyst as a outcome of gray softening Macroscopically: there is a conical infiltrate of purpleblue colour with yellowgreenish apex, which protrudes under the 67. During the histological examination of the fragment of surface of the skin. Diagnose it: cervix it was found out the area of inflammatory infiltration A. Furuncle. with involvement of walls of small vessels. The infiltrate B. Phlegmona. was composed of plasma cells, lymphocytes, epithelioid C. Abscess. cells and areas of sclerosis and hyalinosis. What disease D. Fistula. does such picture characterize? E. Empyema. A. Syphilis. B. Tuberculosis. 73. During the histological examination of the biopsy the C. Leukoplakia. pathologist has found out granulomas within the livers. D. Erosion of cervix. They contain mainly Tlymphocytes and epithelioid cells, E. Condyloma. and solitary giant Langhan’s cells. In the center of granulomas there was a small area of caseous necrosis. 68 During the histological examination of the biopsy it was What pathologic process is characterized by those changes? found out the granulomas composing of lymphocytes, A. Proliferative inflammation. epithelioid cells, plasma cells, macrophages with foamy B. Coagulative necrosis. pale cytoplasm (Mikulicz’s cells), a lot of hyaline balls. C. Liquefactive necrosis. What disease do you think about? D. Neoplasm. A. Rhinoscleroma. E. Exudative inflammation. B. Leprosy. C. Syphilis.
  • 6. 74. A 5yearsold girl suffered from diphtheria. She has died E. Defect of telomere Хchromosoma in three days because of asphyxia caused by diphtheria’s croup. During the autopsy it was found out, that mucous 83. During the autopsy the pathologist has marked: the membranes of larynx, trachea and bronchi were thickening, expressed linear shrinkage and thickening of extremities swelling, dull, covering by grayish membranes, which with formation on a skin of large tucks. The head is easily came off. What process did develop in the larynx? increased, nose is saddle, the oral cavity is semiopen, A. Croupous inflammation. tongue is thick, neck is short, bodies of spondyles are B. Serous inflammation dwarfed, hypoplasia of thorax is combined with hypoplasia C. Diphteritic inflammation of lungs. Which inherent malformation of a musculoskeletal D. Proliferative inflammation system is it characteristic for? E. Suppurative inflammation A. Lethal micromelia B. Chondroplasia 75. As a result of a microscopic examination of the C. Imperfect bone formation fragment of the skin the granulomas were found out; they D. Inherent marble bones composed of a lot of macrophages with a few amount of E. Inherent Oppenheim myatonia lymphocytes and plasma cells. Besides large macrophages with fat vacuoles occurred, the microorganisms were 84. In the autopsy of 48yearold man [miner which got packed in those vacuoles as “cigars in the box” (Virchow’s hematite] the pathologist has found out increased brownred cells). Granulation tissue had a good vascularization. What dense lungs. Microscopic picture is: a moderate disease are such granulomas characterized for? pneumosclerosis, submiliary and miliary nodules, which A. Leprosy consist of dust cells with spots (positive reaction to iron). In B. Tuberculosis. lymph nodes there are a lot of dust and considerable diffuse C. Syphilis. sclerosis. Which professional disease is it? D. Rhinoscleroma. A. Red siderosis E. Actinomycosis. B. Black siderosis C. Aluminosis 76. During the microscopic examination of bioptic fragment D. Berilliosis of the skin the granulomas were found out containing E. Anthracosis epithelioid cells, surrounded with Tlymphocytes. Between the epithelioid cells the solitary giant polynuclear 85. In the autopsy of a body of the miner, which got Langhan’s cells located. There were areas of caseous hematite, the pathologist has found out: black lungs, similar necrosis in the center of some granulomas. Blood vessels on lungs at an anthracosis. Microscopically: moderate were absent. What disease do such changes characterize? pneumosclerosis, submiliar and miliar nodules, which A. Tuberculosis. consist of dust cells with spots [positive reacton to iron]. In B. Syphilis. lymph nodes there are a lot of dust and considerable diffuse C. Leprosy. sclerosis. Your diagnosis? D. Rhinoscleroma. A. Black siderosis E. Hodgkin’s disease. B. Red siderosis C. Aluminosis 77. In a biopsy of a сervix uteri of a26yearold women the D. Berilliosis diagnosis following was established: pseudoerosion. What E. Caplan syndrome microscopical changes has the pathologist revealed? A. Local changes of a stratified squamous epithelium 90. A 45yearold man has died because of on single-layer prismatic one pulmonarycardiac insufficiency. In autopsy the pathologist B. Cellatypia of an epithelium of an mucosal epithelium has found out croupous pneumonia in lower lobe of left C. Keratinization of an epithelium of an mucosal lung, 350ml of greenishyellow fluid in the left pleural epithelium cavity. Microscopically it contained many neutrophils. Call D. “Carcinomatous pearls” the complication of pneumonia E. Local inflammation and necrosis in mucosa A. Empyema of pleura B. Fibrinous pleuritis 80. Inheritable predilection to development of lung C. Pneumothorax emphysema is connected with genetic disturbance, which D. Hydrothorax one shows in decreasing production of: E. Hemothorax A. Alpha1antitrypsin B. Creactive protein 98. A 60yearold miner has died because of chronic C. Antistreptolysin 0 pulmonarycardiac insufficiency. In autopsy the pathologist D. Hyaluronidasa has found out: areas of dystelectasis, induration, E. Complement pneumosclerosis in the both lungs, local emphysema in the apex. Cut surface was slatyblack. Diagnose this disease. 82. Gangliosidlipidosis is characterized by deficiency of A. Anthracosis enzyme alpha –hexoaminidaze and accumulation of B. Silicosis gangliosides in lysosomas of nervous cells.It is connected C. Talcosis. with genetical defect, which is determined as: D. Asbestosis A. Defect of one gene E. Aluminosis. B. Translocation of a gene C. Defects of several genes 108. In 56 year –old patient has suffered from D. Trisomy of 21 chromosoma bronchoectatic disease and hemoptysis, the edema of face
  • 7. and waist have appeared. The protein (33 mg/l) was found A. Sequestrum in urine. Pulmonary hemorrhage was the cause of patient’s B. Dry gangrene death. In autopsy: enlargement of kidneys was found; the C. Wet gangrene kidneys were densed with lardaceus surface of section. D. Infarction Histologically: the deposition of homogenous eosinophilic E. Caseous necrosis masses colored with Congo red and given of metachromasia with methyl violet color in glomeruli and canals were 117. The 46year old woman has suffered from rheumatic found. What pathological process took place in the patient? fever with combined mitral heart disease. In autopsy the 1. Amyloidosis leaflets of mitral valve are thicken, intertwisted and have B. Grainish degeneration; stony dense. What pathological process has led to these C. Fatty degeneration; changes? D. Mucoid degeneration; A. Amyloidosis; E. ;Hyalinosis; B. Hyaline changes; C. Dystrophyc calcification; 111. In 45year old patient died from sudden cardiac death D. Metabolic calcification; the symmetrical type of adipose heart of third degree; the E. Metastatic calcification; rupture of right ventricle’s wall with hemopericardium and redundant accumulation of fat under epicardium were found 118. Patient is suffered from cholera, which is during with in autopsy. Microscopically: the adipose tissue grows from dehydratating, cyanosis and convulsions. In result of epicardium into myocardium with atrophy of fibers of massive infusion therapy the exicosis is diminished, but muscle. What process is more probable? anuria is remained. Patient is dead of uremia. Choose A. Adipose heart; correct position for this case. B. Hypertensive disease; A. Necrotic nephrosis with cortical necrosis takes place C. Ischemic heart disease; in the kidneys, D. Fatty degeneration of myocardium; B. Choleral typhoid is developed, E. Acute myocardial infarction; C. Development of uremia is connected with acute glomerulonephritis, 112. The 48yearold patient, suffered from fibrouscavernous D. Fibrinouse colitis is found in autopsy, tuberculosis, has complained of weakness, reduction of E. Exicosis is due to action of virus exotoxin. daily secretion of urine, edema of the body and extremities and increasing of blood pressure to 180/90. The increasing 119. Please, choose incorrect statement. of protein, presence of hyaline and grain cylinders and A. Degeneration is characterized by disorder of tissue erythrocytes were found under analysis of urine. The patient nutrition and in every time lead to distraction of has died in a month because of the insufficiency of kidney. nucleus and whole cell In autopsy, the enlargement of the heart and "lardaceus" B. There are extracellular and intracellular accumulations kidneys with mass of more than 240 g were found. What is C. Pathological calcification may be in two forms: the complication of fibrosecavernouse tuberculosis? dystrophic and metastatic calcification A. Аmyloidosis; D. Degenerations may be caused by hereditary factors B. Glomerulonephritis; E. Degenerations can appear in one organ or in the whole C. Nephrotic syndrome; body D. Pyelonephritis; E. Nephrosclerosis; 120. In autopsy: enlargement of kidneys was found; the kidneys were densed with lardaceus surface of section. 114. In patient with jaundice the following data were Histologically: the deposition of homogenous eosinophilic established: in serum the increasing of bilirubin because of masses colored with Congo red and given of metachromasia the unconjugated form; in faeces and urine increasing of with methyl violet color in glomeruli and canals were stercobilin; the level of conjugated (direct) bilirubin in found. What pathological process took place in the patient? serum is normal. What type of jaundice takes place? A. Amyloidosis A. Haemolytic jaundice; B. Grainish degeneration; B. Jaundice of newborns; C. Fatty degeneration; C. Parenchymatous (hepatic) jaundice; D. Mucoid degeneration; D. Gilbert’s disease; E. ;Hyalinosis; E. Mechanical (posthepatic) jaundice; 121. Choose one incorrect statement: 115. In autopsy of the 83yearold patient’s body, has died A. The mechanism of decomposition is a base of because of stomach cancer, the heart and the liver were genetic storage diseases; diminished in size, condensed, with brown color. What B. Decomposition (phanerosis) – disintegration of pathological process took place in these organs? membranous structures of cells and intercellular A. Sclerosis; matrix; B. Mesemchymal degeneration; C. Infiltration – redundant accumulation (deposition) of C. Parenchymal degeneration; metabolites into the cells and intercellular matrix; D. Necrosis; D. Perverted synthesis – synthesis of abnormal substances E. Brown atrophy in the cells and tissues; E. Transformation – formation of one type of metabolism 116. Fragment of dead tissue, which can’t be autolized, products from common initial substances for protein, replaced by connective tissue and which is localized among fats and carbohydrates. alive tissue is named…
  • 8. 122. Intracellular accumulation of fat in liver is lowerlobar pneumonia with expectoration of mucus with characterized by every statements, except one: pus. In autopsy in 910 segments of the right lung the cavity A. Fat is accumulated in extracellular spaces with dense walls filled with purulent masses, was found. B. There are small fat vacuoles in cytoplasm around the The whitish path comes from the cavity toward the radix of nuclei of hepatocytes the lung. Microscopically in was established that the cavity C. Liver enlarged and becomes yellow is divided from saved lung tissue with thin membrane, D. Its consistency is soft and greasy which consists of two layers: internalgranulation tissue, and E. Fat is stained by special stain Sudan 3, and becomes external –connective tissue. What diagnosis is more orangered color probable? A. Chronic abscess; 123. A 55 year old man has died after chronic B. Pulmonary gangrene; glomerulonephritis, chronic renal insufficiency. In the C. Acute pulmonary abscess; autopsy the pathologist has found out characteristic changes D. Chronic pneumonia; in kidneys for this disease, also fibrinous pericarditis, E. Bronchoectatic disease pleuritis, bronchitis. Call the cause of the fibrinous inflammation in serosal and mucosal layers. 150. Cyanosis, enlargement of the liver, edema of the low A. Uremia. extremities as a result of insufficiency of the right ventricle B. Hypolipidemia of the heart were found in a patient. What is the cause of the C. Hyperlipidemia development of such insufficiency? D. Arterial hypertension A. Hypertension of the small circle of the blood E. Arterial plethora circulation B. Functional shunts in lungs 140. During autopsy of the patient died from chronic C. Increasing of the venous pressure cardiac insufficiency the enlarged dense rusty colored lungs D. Hypercathecholemia with growth of gray color connective tissue around bronchi E. Cardiogenic cirrhosis of the liver and vessels were found. How can we call this process in lungs? 153. In 53 yearold patient, who has suffered by А Brown induration of the lung bronchoectatic disease and hemoptysis, the edema of face В Hemorrhagic pneumonia and waist have appeared. The protein (33 mg/l) was found С Interstitial emphysema in urine. Pulmonary hemorrhage was the cause of patient’s D Chronic obstructive emphysema death. In autopsy: enlargement of kidneys was found; the E Primary idiopathic emphysema; kidneys were dense with lardaceous surface of section. Histologically: the deposition of homogenous eosinophilic 143. In 77yearold patient suffered with atherosclerosis the masses colored with Congo red and given of metachromasia pain has appeared in the right foot. The foot is enlarged in with methylviolet color in glomeruli and canals were found. size, its skin has black color and is macerated; the What pathological process took place in the patient? demarcation line is not clear. What pathological process A. Amyloidosis; takes place in a patient? B. Fatty degeneration; A. Wet gangrene C. Mucoid degeneration; B. Coagulative necrosis D. Grainish degeneration C. Sequestrum E. Hyalinosis D. Dry gangrene E. Noma 155. In the case of rheumatic heart disease, pericarditis is characterized by: 145. A 65 year old patient, who suffered from A. “Shaggy heart” (“cor villosum”) arteriosclerosis, has been hospitalized in surgical B. “Cor pulmonale” department because he had purulent peritonitis. Thrombosis C. “Tiger heart” of mesenteric arteries was found during operation. What is D. Fatty change the most probable cause of peritonitis? E. “Bull heart” A. Hemorrhagic infarction B. Angiospastic ischemia 156. In a 45yearold patient the ulcerativenecrotic damage of C. Angioneurotic edema the mucosa of the oral cavity takes place; also the spread D. Stasis lymphadenopathy, slight spleno and hepatomegaly, diffuse E. Chronic congestion hemorrhages in the skin and mucous membranes were found out. In blood analysis the increasing of leukocytes (to 148. Under microscopic investigation the postinfarction 100 10 per 1 ml) at the account of lymphoblasts takes place. cardiosclerosis has been found out. Around cardiosclerotic What diagnosis is more probable? area myocardiocytes were enlarged in size and had large A. Acute lymphoblastic leukemia hyperchromic nuclei riched in DNA. What process taking B. Chronic myelocytic leukemia place in myocardiocytes is more probable? C. Chronic lymphocytic leukemia A. Regenerative hypertrophy D. Plasmocytosis B. Physiologic regeneration E. Acute promyelocytic leukemia; C. Complete reparative regeneration D. Pathologic regeneration 159. Chronic venous congestion of organs, hypertrophy of E. Hypertrophy because of increased workload the left heart ventricle with cardiosclerosis, volumetrical white-yellowish plaques in intima of aorta with 149. 56yearold patient has suffered from rightside atheromatous masses in the center, which are deeper in the
  • 9. wall, are found in the result of autopsy of 70year man, who E. Perineural died of heart insufficiency and who suffered from angina pectoris, hypercholesterinemia and obesity. Which 170. At the autopsy of 60 yearold male patient, in the pathological process is more possible in aorta? myocardium of the front wall of the left ventricle of the A. Atheromatosis heart a gray dense focus of 5 х 4 cm, of irregular shape with B. Lipoidosis clear borders, fibrous structure has been found out. What C. Arteriolosclerosis pathological process in the myocardium has revealed the D. Liposclerosis pathologist? E. Calcification A. Postinfarctional cardiosclerosis B. Diffuse smallfocal myocardiosclerosis 161. All statements are correct in the relations of hyalinosis, C. Myocarditis except one: D. Petrification of the myocardium A. The hemorrhage and hemoliysis play important role E. Infarction in the hyalinosis formation B. Hyaline is accumulated in the connective tissue and 171. In 40yearold patient, the tumor, which grew under skin looks like cartilage of spine was resected. The histologic diagnosis: a lipoma. C. Mainly arterioles and arteries undergo to hyalinosis What principle of the tumors’ classification did the D. Hyalinosis is rare reversible process pathologist use when created his conclusion? E. There are not macroscopical changes of the organs A. Gistogenesis usually, but hyalinosis may lead to deformation and B. Of biochemical features wrinkle of the organs C. Of ultrastructural features D. Of physicochemical features 166. In 60yearold patient, during examination the cancer of E. Macrostructure of an organ a prostate gland with metastases in the lower parts of the column, pelvic bones and pathological fracture of a femur 172. At histologic research of a biopsy specimen from an was found out. What pathological process is a basis of the auricle of a heart of a patient with rheumatic disease the development of metastases? foci of a mucoid swelling, fibrinoid necrosis of a connective A. Tissue embolism tissue has been found out. What immune response has B. Bacterial embolism developed in tissues of the auricle of the heart? C. Airembolism A. Hypersensitivity of an immediate type D. Embolism by foreign matters B. Hypersensitivity of a delayed type E. Necrosis of tissue C. Reaction of the transplantative immunity D. Normergic reaction 167. In patient with chronic cystitis during the investigation E. Exudative reaction of biopsy of a mucosa of urinary bladder the foci of a squamous nonkeratinized epithelium are found out. It can 178. Call a kind of an infarct according to macroscopic be a sign of development of: signs, which is characteristic in myocardium. A. Metaplasia A. White with a hemorrhagic halo B. Leukoplakia B. Hemorrhagic C. Hyperplasia C. Anemic D. Dysplasia D. Mixed E. Hyperkeratosis E. Red 168. During the laparotomy in 49 yearold male patient, the 179. What type of exudate appears in pericardium cavity at tumor has been found out in the field of a sigma with a rheumatic pericarditis? growth through all its layers and an occlusion of the lumen A. Serous-fibrinous of an intestine. The biopsy has been taken and colonostoma B. Putrefactive has been overlapped. The clinical diagnosis after operation: C. Hemorrhagic a cancer of sigma. What kind of tumor is growth in relation D. Purulent to tissues? E. Catarrhal A. Infiltrative B. Expansive 180. At autopsy of a patient died because of a cerebral C. Endophytic hemorrhage, strongly enlarged dense and anemic kidneys D. Exophytic ( size :6х3х2см weight 60.0,) with a uniformly E. Multicentric smallgranulated surface and with uniformly thinned cortex on a cutsurface have been found out. The changes in kidney 169. In 36 yearold patient a dark brown color patch on skin are a sign of: of a dextral foot has been resected. The histologic research A. Of arteriosclerotic nephrosclerosis has not been carried out. In 5 months in dextral inguinal B. Of atherosclerotic nephrosclerosis area a conglomeration of lymph nodes has appeared. At C. Secondary shrinkage of kidney histologic research of one of them the metastasis of D. Gouty (podagric) kidneys melanoma has been found out. What pathway of spreading E. Amiloidshrinkaged kidneys (“lardaceous kidneys”) of tumor takes place? A. Limphogenous spread 183. A 50yearold man had rheumatic mitral disease. He has B. Hematogenous spread arrived in clinic with sings of active rheumatism. On a C. Anaplastic background of increasing cardiovascular insufficiency has D. Mixed developed lefthand hemyplegia. The patient has died. What
  • 10. changes of a brain have caused development lefthand E. The type V hemyplegia? A. Infarction of a brain 207. The 35yearold female was admitted into the hospital B. Hematoma. with nephrolithiasis. Anaphylactic shock was appeared after C. Atherosclerosis of brain’s vessels. intravenous injection of contrast medium. Which cells take D. Edema of a brain. part in development of this reaction? E. Meningitis. A. Tissue’s basophils B. Fibroblasts 187. At histologic examination of tuberculoma resected C. Epitheliocytes from right lung a necrosis in center is found out. Call a kind D. Myocytes of the necrosis. E. Neutrophils A. Caseous B. Fatty 208. Histologic investigation of thyroid gland has showed C. Liquefactive destruction and atrophy of follicles, diffuse lymphoid D. Coagulative infiltration with formation of lymphoid follicles in the E. Gangrene stroma. Call the group of diseases with respect to this thyroiditis. 194. A woman, which was in a bed in forced position for a A. Autoimmune specific long time, has died because of a tumoral dissemination B. Autoimmune nonspecific (multiple metastases and carcinomatosis of a peritoneum) of C. Bacterial mucinous cystadenocarcinoma. During the autopsy the D. Infectiousallergic large areas of necrosis of the skin and underlying soft E. Viruses infectious tissues of sacral part were found out. Call a kind of a necrosis. 209. Ultrastructural investigation of tissue’s biopsy has A. Bedsore showed considerable increase of lyzosomes in the B. Infarct cytoplasm of macrophages within the inflammative C. Sequester infiltrate. What is connected with appearance of this D. Caseous necrosis morphologic process? E. Waxylike (Zenker’s) necrosis A. Activation of phagocytosis B. Proliferation of cells 198. The examination of the child with measles showed the C. Torpid phagocytosis nonclear border edematous fluctuated areas of redblack D. Activation of apoptosis color in the soft tissues of the cheeks and perineum. What E. Torpid apoptosis complication did develop in the child? A. Wet gangrene (noma) 210. In biopsy of stomach in patient with autoimmune B. Dry gangrene gastritis it was found out: infiltration by lymphocytes and C. Gas gangrene macrophages in mucous layer. Which type of D. Bedsore hypersensitivity is connected with these morphologic E. Trophic ulcer changes? A. The type IV 202. The presence of the grains of chromatin in a focus of B. The type II caseous necrosis is a manifestation of: C. The type V A. Karyorhexis D. The type I B. Karyolysis E. The type III C. Karyopyknosis D. Mitotic activity of nuclei 211. In biopsy of lymph node it was found out a lot of E. Apoptosis lymphoid follicles with large centers of duplication, increasing of mitoses. Which process is characterized by 205. In 77yearold patient suffered with atherosclerosis the these morphologic changes? pain in the right foot has appeared. The foot is enlarged in A. Antigenic stimulation with follicular size, its skin has black color and is macerated; the hyperplasia demarcation line is not clear. What pathological process B. Atrophy of lymphoid tissue takes place in the patient? C. Lymphosarcoma A. Wet gangrene D. Hodgkin’s disease B. Coagulative necrosis E. Metastases of cancer C. Sequestrum D. Dry gangrene 212. It was found stopping of breathing and blood E. Noma circulation in patient after traffic accident. Resuscitation was successful. Call the that condition of patient. 206. The 30yearold man has had for two months A. Clinical death lacrination, pruritic palpebras, rhinitis with mucus. All B. Social death symptoms disappeared after treatment by desensebilizators. C. Biological death What type of hypersensitivity occurred in patient? D. Physiologic death A. The type I E. Agony B. The type II C. The type III 213. Call the cells of the immune system that secrete D. The type IV immunoglobulins:
  • 11. A. Plasma cells Goodpasture’s syndrome. Which pathologic process is B. Macrophages characteristic for this syndrome? C. Tlymphocytes A. Autoantibodies to basement membrane of the D. Dendritic cells lungs and kidneys E. Natural killer cells B. Autoantibodies to mitochondrias of the lungs and kidneys 214. Which cells are injured in patient with HIVinfection? C. Autoantibodies to DNA A. Tcells-helpers D. Cytotoxic reaction against epithelium of renal B. Blymphocytes tubules C. Natural killer cells E. Appearance of immune complexes in glomeruli of D. Macrophages kidneys and lungs E. Eosinophils 225. The patient was remove the firmly – elastic, fixed, 215. The 35yearold man has received prolonged painful tumor, it is localized in femor. At pathomorphologic immunosupressive therapy after transplantation of kidney. research it was found out: yellow tumor without the He has died because of generalized cytomegaly. Call the capsule, greyish with the locuses of necrosis and kind of immunodeficiency in patient. hemorrhages; histologically is marked the expressed A. Medicamentous polymorphism of cells. What is your diagnosis? B. Xradiating A. Liposarcoma C. Bacterial B. Rhabdomyosarcoma D. Primary C. Angiosarcoma E. Ecological D. Lipoma E. Malignant synovioma 216. The 30yearold patient with transplanted kidney has received prolonged immunosupressive therapy and he has 226. Under microscopic examination of the increased died because of intoxication. Microscopic examination cervical lymph node it was found out: the absence of its showed giant cells with large nuclei encircled by ringslike structure and the lymphoid follicles, infiltration by spherical brightening, which looked as “owleye”, located in the cells with narrow cytoplasm. It is known from the clinical kidneys, liver, pancreas, lungs. Call this disease. dates, that other groups of lymph nodules, and lien and liver A. Cytomegalovirus infection are increased also. What disease it is necessary to think B. Tuberculosis about? C. Syphilis A. Lymphoid leukosis D. Leprosy B. Lymphogranulomatosis E. Bubonic plaque C. Lymphosarcoma D. Myeloid leucosis 217. In biopsy of the lymph node it was found out E. Myeloma granuloma with epithelioid and giant cells. Call the origin of epithelioid cell’s development. 227. During autopsy of the body of the 30yearsold man has A. Monocytes been determined the increased lien (weight 900,0), B. Pericytes increased liver (weight 4000,0), increased lymph nodes. C. Neutrophils Bone marrow of a diaphysis of a femur was red color. D. Lymphocytes Microscopically: infiltration of the portal tract of liver by E. Epithelium cells undifferentiated hemopoietic cells with the spherical shape and narrow cytoplasm. What disease it is possible to think 222. In biopsy of transplantanted kidney it was found out: about? diffuse infiltration of stroma by lymphocytes, plasma cells, A. Chronic lymphoid leucosis lymphoblasts, plasmablasts, necrotic arteriitis. Which B. Chronic myeloid leucosis pathologic process was appeared in organ? C. Generalized form of lymphogranulomatosis A. Immune mutilation D. Acute myeloblastic leucosis B. Acute glomerulonephritis E. Acute lymphoblastic leucosis C. Ischemic infarction D. Tuberculosis 228. At research of a biopsy of the increased cervical lymph E. Acute pyelonephritis node it were found: the absence of structure of lymph nodes, infiltration by proliferated lymphocytes, single 223. Pest’s pneumonia and croupous pneumonia in stage of Shternberg’s cells. What do these changes testify about? red hepatization may be differentiated due to presence of A. Lymphogranulomatosis with predominance of following elements in the lungs: lymphoid tissue, A. Fibrin B. Lymphogranulomatosis with suppression of lymphoid B. Carnification tissue C. Macrophages C. Mixed cellular variant of lymphogranulomatosis D. Red blood cells D. Nodularsclerotic variant of lymphogranulomatosis E. Edema E. Lymphosarcoma 224. In the 30yearold woman it was found: cough, sputum 229. In biopsy of the increased cervical lymph node of a 14 with blood, fever, increased blood pressure, decreased urine yearold girl the pathologist has found out: the absence of output, edema of low extremities. All symptoms have structure of lymph nodes, the absence of lymphoid follicles, developed for 6 weeks. Diagnostic renal biopsy showed foci of necrosis and sclerosis, cellular polymorphism.
  • 12. There are lymphocytes, eosinocytes, atypical multinuclear distributed irregular. Described morphological features are cells of the large sizes and uninuclear cells of large sizes characteristic for…: also. Call this disease. a. Fibroma A. Lymphogranulomatosis B. Rabdomyoma B. Acute lymphoid leucosis C. Papilloma C. Chronic lymphoid leucosis D. Polyps D. Berkit’s lymphoma E. Lipoma E. Diffuse nonhodgkin’s lymphoma 235. In an autopsy of the 60yearold woman pathologist 230. A 14 yearold boy was found out: the increase of lower has found out: tumor of uterine with growth surrounded third of femur, local hyperemia, strengthening venous tissue, enlarged regional lymph nodule, hemoperitoneum. structure in a zone of pathological process. Ro: injury of Histologically: cellular polymorphism and tissue atypism supperficial layer of a femoral bone, joint are saved. In in the endomtrium, necrosis and hemorrhages, biopsy was found out the accumulation of atypical metastasises of adenocarcinoma of lymph nodules. What osteoblasts with multiple mitosises and foci of the are secondary appearances of tumors? anomalously formed osteal girders with an invasion in 1. Necrosis and hemorrhage surraunded tissues. Call this disease. B. Lymphogenous metastasises 1. Osteogenic a sarcoma C. Hematogenous metastasises B. Chondrosarcoma D. Cellular polymorphism C. Fibrous dysplasia E. Tissue atypism D. Osteoid osteoma E. Osteoblastoclastoma 236. In young 18yearold woman the sharp pain during swallowing, enlargement of lymph nodes of the neck and 231. A 40 yearold man has been suffered by a chronic increasing of temperature to 39 degrees of Centigrade have bronchitis has died because of cachexy. During the autopsy appeared. The whitishyellowish pellicles have appeared on it was found: in a lumen of bronchuses the endophytic the surface of tonsil’s mucosa. After separation of these growth of light grayish soft tissue. Under the microscopic pellicles the ulceration of tissue was formed. The state of examination in a wall of a bronchus among growths of an health of the patient has become progressively worse. She atypical squamous epithelium were found the keratinous has died in 8 days of the disease with signs of progressive pearls. What is your diagnosis? heart insufficiency. What histological changes in 1. Squamous carcinoma with a keratinization cardiomyocytes can be found? B. Squamous carcinoma without a keratinization A. Fatty degeneration; C. Adenocarcinoma B. Mucousal degeneration; D. Undifferentiated carcinoma C. Hydropic degeneration; E. Apudoma D. Balloon degeneration; E. Hyalinedroplets degeneration. 232. At intraoperation biopsy of a female brest it was found out: the concentric growth intralobular connective 237. A 55yearold woman has had fever and dyspnea for tissue around of smallsized duct covered by a singlelayer over a month along with a 2 kg weight loss. On physical epithelium Your presumable diagnosis. examination her temperature is 37.8 C. A chest radiograph A. Fibroadenoma shows a reticulonodular pattern along with prominent hilar B. Intraductal papilloma lymphadenopathy. A transbronchial lung biopsy is C. Lobular carcinoma in sity performed, and microscopic examination shows no viral D. Noninvasive intraductul carcinoma inclusions, no fungi, no acid fast bacilli, and no atypical E. Mastitis cells. Which of the following diseases is she most likely to have? 233. At gastroscopical research of patient in area of A. Silicosis lesser curvature of the stomach it was found out the B. Sarcoidosis exophytic formation (1,5 cm in diameter). In biopsy C. Asbestosis from form it is marked : the growth of atypical D. Tuberculosis glandular structures of different shape and size E. Usual interstitial pneumonitis (stretched, round, with irregular outlines, small or large) into mucosa and submucosa; the epithelial 238. A 60yearold man has had a cough without production cells are atypical too: their nuclei are polymorphous of much sputum for the past week. On physical examination with various maintenance of chromatin and mitoses. he is afebrile. There are decreased breath sounds at the right What is your diagnose? lung base. A chest radiograph reveals an area of A. Adenocarcinoma of stomach consolidation in the right lower lobe. He is given antibiotic B. Squamous carcinoma of stomach therapy, but a month later the radiographic picture has not C. Polyp of stomach changed, and his cough continues. A bronchoalveolar D. Fibrous carcinoma (Skyrrus) lavage is performed and yields atypical cells along with E. Chronic ulcer scattered alveolar macrophages. Which of the following is the most likely diagnosis? 234. In the skin it was found out well –encapsulated firm A. Mycoplasma pneumonia node. On the cut it is showed whitish laminated tissue. B. Bronchioloalveolar carcinoma Histologically: the welldifferentiated connective tissue, the C. Sarcoidosis bands of fibers lay in different directions, vessels are D. Pulmonary infarction E. Silicosis
  • 13. test is negative. By which of the following immunologic 239. Following a vehicular accident with blood loss leading mechanisms is her disease most likely produced? to prolonged, severe hypotension, a 30yearold man is A. Local immune complex formation intubated and placed on a ventilator. He has progressively B. Interleukin release from macrophages decreasing oxygen saturations despite increasing PEEP and C. Binding of antireceptor antibody FI02 of 100%. He remains afebrile. He dies 3 days later. At D. Mast cell degranulation autopsy, the lungs show diffuse hyaline membranes in E. Crossreactivity with tissue antigens alveoli, thickened alveolar walls, and many alveolar macrophages but few neutrophils. Which of the following 252. A bee sting is suffered by a 28yearold man, and his pulmonary diseases most likely complicated his course? wife searches frantically for the medical kit with the A. Bronchopneumonia injectible epinephrine. Which of the following immunologic B. Chronic bronchitis mechanisms are they trying to prevent? C. Bronchiectasis A. Local immune complex formation D. Viral pneumonia B. Interleukin release from macrophages E. Diffuse alveolar damage D. Systemic anaphylaxis C. Binding of antireceptor antibody 240. A 66yearold man has had increasing malaise for the E. Complement activation past year. On physical examination auscultation of the chest reveals a friction rub. Laboratory studies show a serum urea 253. A 9yearold boy has a sore throat. A throat culture nitrogen of 100 mg/dl and creatinine of 9.8 mg/dl. Which of grows group A hemolytic streptococcus. 17 days later the following forms of pericarditis is he most likely to develops darkcoloured urine. A renal biopsy was have? performed. On immunofluoresence staining the biopsy A. Fibrinous shows granular deposition of IgG and complement around B. Hemorrhagic glomerular capillary loops. Which of the following immune C. Purulent hypersensitivity mechanisms is most likely responsible for D. Serous this pattern of findings? E. Constrictive A. Type I B. Type II 244. A 45yearold man was rushed to the hospital following C. Type III the sudden onset of an episode of crushing substernal chest D. Type IV pain. He receives advanced life support measures. His E. Type V course was marked by intractable cardiogenic shock and he died 4 days later. At autopsy, a large transmural 254. A 48yearold man has had a chronic cough with fever anterolateral area of coagulative necrosis was found in the for 2 months. On physical examination his temperature is anterolateral wall of the left ventricle. Which of the 37.9 C. A chest radiograph reveals a diffuse bilateral following microscopic findings is most likely to be present? reticulonodular pattern. A transbronchial biopsy is A. Fibroblasts and collagen performed. On microscopic examination of the biopsy there B. Granulation tissue are focal areas of inflammation containing epithelioid C. Necrotic muscle and neutrophils macrophages, Langhans giant cells, and lymphocytes. D. Granulomatous inflammation These findings are most typical for which of the following E. Diffuse chronic inflammation immunologic responses? A. Type I hypersensitivity 250. A 41yearold man has a history of drinking 1 to 2 liters B. Type II hypersensitivity of whisky per day for the past 20 years. He has had C. Graft versus host disease numerous episodes of nausea and vomiting in the past 5 D. Polyclonal Bcell activation years. He experiences a bout of prolonged vomiting, E. Type IV hypersensitivity followed by massive hematemesis. On physical examination in the emergency room, he has vital signs with 255. Twelve hours after going on a hike through dense T 36.8 C, P 110, RR 22, and BP 80/40 mm Hg. His heart foliage, a 40yearold man notices a slightly raised and tender has a regular rate and rhythm with no murmurs and his irregular reddish rash on one forearm that was not covered lungs are clear to auscultation. There is no abdominal by clothing. This rash gradually increases in intensity for 2 tenderness or distension and bowel sounds are present. His days and then fades after two weeks. Which of the stool is negative for occult blood. Which of the following is following forms of hypersensitivity is most likely the most likely diagnosis? demonstrated in this patient? A. Esophageal stricture A Type I hypersensitivity B. Esophageal laceration (MalloryWeiss syndrome) B Type II hypersensitivity C. Esophageal pulsion diverticulum C Type III hypersensitivity D. Barrett esophagus (metaplasia with gastric mucosa) D Type IV hypersensitivity E. Esophageal squamous cell carcinoma E. Type V hypersensitivity 251. A 45yearold woman has experienced progressive, 256. The examination of ovarium tumor was performed. On increasing muscular weakness, particularly toward the end gross inspection of the mass, that was surgically excised, of the day, over the past 2 months. She does not have the surface of the mass is smooth, is not adherent to arthralgias or myalgias. On physical examination her motor surrounding pelvic structure, and is cystic and filled with strength goes from 5/5 to 4/5 with repetitive movement of hair on sectioning. On microscopic examination there is extremities. A chest CT scan reveals an anterior mediastinal squamous epithelium, tall columnar glandular epithelium, mass. Laboratory studies show that her antinuclear antibody
  • 14. cartilage, and fibrous connective tissue. Which of the D Increased nuclear/cytoplasmic ratio following neoplasms is she most likely to have? E Expansive type of growth A Teratoma B Choristoma 262. A clinical study of patient with pharyngeal infections C Hamartoma is performed. Patient experienced fever and chills. On D Myxoma physical examination, the most common finding is a E Mesothelioma pharyngeal purulent exudate. Which of the following types of inflammation did these patient most likely have? 257. A 44yearold woman who has had multiple sexual A. Acute inflammation partners for the past 30 years has an abnormal Pap smear B. Granulomatous inflammation with cytologic changes suggesting human papillomavirus C. Abscess formation infection. Without treatment, she is most likely to develop D. Resolution of inflammation which of the following lesions? E. Chronic inflammation A Squamous cell carcinoma B NonHodgkin's lymphoma 263. An empyema is a kind of inflammation as: C Kaposi's sarcoma A Purulent inflammation D Adenocarcinoma B Serous inflammation E Leiomyoma C Fibrinous inflammation D Chronic inflammation 258. A 41yearold woman has an exophytic 3 cm mass E Granulomatous inflammation involving the ectocervix. Pap smear testing is performed; she has never had a previous Pap smear. Cytologic changes 264. A 45yearold man has had a fever and dry cough and that are seen in Pap smears reveals squamous cell his temperature is 38.5 C during last 3 days. A chest carcinoma. Her serum glucose is 157 mg/dL. She has been radiograph shows a right pleural effusion. A right a commercial sex worker in the past. Which of the thoracentesis is performed. The fluid obtained has a cloudy following is the most likely risk factor for her cervical appearance with a cell count showing 5500 leukocytes per carcinoma? microliter, 98% of which are neutrophils. Which of the A Human papillomavirus infection following terms best describes his pleural process? B Diabetes mellitus, type II A Purulent inflammation C Heavy cigarette smoking B Serous inflammation D Pelvic inflammatory disease C Fibrinous inflammation E Previous cancer chemotherapy D Chronic inflammation E Granulomatous inflammation 259. A 27yearold woman in excellent health has a 2 cm firm, rounded mass is palpable beneath the skin of the left 278. What type of exudates appears in pericardium cavity at forearm. She has no difficulty using the arm and there is no a rheumatic pericarditis? associated pain with the mass, either in movement or on A. Putrefactive palpation. The overlying skin appears normal. The mass B. Serousfibrinous does not change in size over the next year. Which of the C. Hemorrhagic following neoplasms is she most likely to have? D. Purulent A Lipoma E. Catarrhal B Metastatic carcinoma C Melanoma 284. A 90yearold woman has pneumonia. Two weeks later D Rhabdomyosarcoma a chest radiograph reveals a 3 cm rounded density in the E Leiomyosarcoma right lower lobe whose liquefied contents form a central airfluid level and has a capsule. Which of the following is 260. A 35yearold woman had a firm nodule palpable on the the best description for this outcome of her pneumonia? dome of the uterus six years ago recorded on routine A Abscess formation physical examination. The nodule has slowly increased in B Hypertrophic scar size and is now appears to be about twice the size it was C Regeneration when first discovered. She remains asymptomatic. Which of D Bronchogenic carcinoma the following neoplasms is she most likely to have? E Progression to chronic inflammation A Leiomyoma B Adenocarcinoma 285. A clinical study is peformed of patients with C Leiomyosarcoma pharyngeal infections. The most typical clinical course D Hematoma averages 3 days from the time of onset until the patient sees E Metastasis the physician. Most of these patients experienced fever and chills. On physical examination, the most common finding 261. A study is performed to analyze characteristics of is a pharyngeal purulent exudate. Which of the following malignant neoplasms in biopsy specimens. The biopsies types of inflammation did these patients most likely have? were performed on patients who had palpable mass lesions A Acute inflammation on digital rectal examination. Which of the following B Granulomatous inflammation microscopic findings are most likely to indicate that the C Abscess formation neoplasm is malignant? D Resolution of inflammation A Invasion E Chronic inflammation B Tissue pleomorphism C Tissue atypia 286. At 38 yearold woman it is observed upperclavicle
  • 15. multiplied lymphatic nodules. During biopsy research in the gastritis has experienced weight loss and nausea for the past lymphatic nodules a metastasis of fingerringcellular cancer 8 months. He does not have vomiting or diarrhea. In the is exposed. What organ cancer most probably did develop antrum of stomach endoscopy reveals an ulcerative mass at the patient? with elevated margins. Patient undergoes gastrectomy, and A Cancer of stomach the gross appearance shows ulcerated lesion with necrosis B Cancer of thyroid in the center that has extended below the submucosa, into C Cancer of lungs the muscularis, and has spread more widely extends into D Cancer of gullet the muscularis propria and beyond. Identify the type of E Cancer of uterus lesions: A. Early gastric carcinoma 289. A 70yearold male with a long history of chronic B. Advanced gastric carcinoma alcoholism has had increasing difficulty with swallowing C. Duodenum carcinoma for the past 2 months. Upper endoscopy reveals an D. Acute gastric ulcer ulcerative midesophageal, 3cm mass that partially occludes E. Chronic gastric ulcer the esophageal lumen. He undergoes esophagectomy, and the gross appearance shows ulcerated lesion with heaped up 237. An upper gastrointestinal radiographic series reveals margins and squamous cell atypia of the esophageal advanced gastric carcinoma in a 53yearold female who has mucosa. Identify the type of lesions: had nausea, vomiting, and midepigastric pain for several A. Leiomyosarcoma months. Ultrasound investigation reveals an solid tumorous B. Squamous cell carcinoma enlargements of both ovaries 6x5 cm mass. Identify the C. Dense collagenous scar type of lesions in ovaries: D. Adenocarcinoma A. NonHodgkin lymphoma E. Thrombosed vascular channels B. Cystadenoma of ovaries C. Virchow's node 230. An upper gastrointestinal radiographic series reveals D. Krukenberg tumors gastric outlet obstruction in a 53yearold female who has had E. Cystadenocarcima of ovaries nausea, vomiting, and midepigastric pain for several months. Upper endoscopy reveals an ulcerated 3x4 cm 238. A 67yearold male with a history of brain hemorrhage mass at the pylorus. The biopsy was performed. Which of has had an episodes of hematemesis at the hospital during the following neoplasms is most likely to be seen on biopsy past 2 weeks after hemorrhage. In the mucous shell of the of this mass? stomach body endoscopy reveals an ulcer 2.5cm in A. NonHodgkin lymphoma diameter, round form, with a brown base. Identify the type B. Neuroendocrine carcinoma of ulcer: C. Squamous cell carcinoma A. Curling's D. Adenocarcinoma B. Virchow's E. Leiomyosarcoma C. Chronic D. Cushing's 231. A 67yearold male with a long history of Barrett E. Krukenberg’s esophagus has had increasing difficulty with swallowing for the past 2 months. In the lower third part of the esophagus 240. During autopsy it was found out croupous pneumonia upper endoscopy reveals an ulcerative esophageal mass that in lower lobe of the left lung and about 500ml of partially occludes the esophageal lumen. Patient undergoes greenishyellow fluid in the pleural cavity. Microscopically esophagectomy, and the gross appearance shows ulcerated pleural liquid contained many neutrophils. Identify the lesion with necrosis in the center. Identify the type of complication of pneumonia: lesions: A. Hemothorax A. Leiomyosarcoma B. Fibrinous pleuritis B. Squamous cell carcinoma C. Pneumothorax C. Dense collagenous scar D. Hydrothorax D. Adenocarcinoma E. Empyema of pleura E. Thrombosed vascular channels 241. 56yearold patient has suffered from rightside 235. A barium swallow is performed in a 44yearold female lowerlobar pneumonia with expectoration of mucus with who has had nausea and vomiting for months. It is known pus. During autopsy in 910 segments of the right lung the that she had a stomach ulcer for many years. cavity with dense walls filled with purulent masses, was Radiographically, there is marked dilation of the stomach found. The whitish path comes from the cavity toward the with "beaking" in the distal portion where marked luminal radix of the lung. Microscopically it was established that narrowing exists. A biopsy of the antrum shows ulcer the cavity is separated from notchanged lung tissue with formation with elevated margins and prominent submucosal thin membrane, which consists of two layers: fibrosis without inflammation, epithelium cell atypism. The internalgranulation tissue, and external – connective tissue. most likely cause for these findings is What diagnosis is more probable? A. Chronic duodenum ulcer A. Chronic abscess B. Chronic gastric ulcer B. Pulmonary gangrene C. Acute gastric ulcer C. Acute pulmonary abscess D. Gastric carcinoma D. Chronic pneumonia E. Duodenum carcinoma E. Bronchoectatic disease 236. A 72yearold male with a long history of chronic